mynclex set 6- 40

¡Supera tus tareas y exámenes ahora con Quizwiz!

The nurse is caring for a client with a balloon tamponade tube in place due to bleeding esophageal varices. The client suddenly develops respiratory distress, and the nurse finds that the tube has been partially pulled out. Which intervention should be the nurse's priority? 1. Contact the health care provider (7%) 2. Cut the tube with scissors (46%) 3. Increase gastric suction level (1%) 4. Place the client in high Fowler position (44%) OmittedCorrect answer 2 46%Answered correctly

A balloon tamponade tube (eg, Sengstaken-Blakemore, Minnesota) is used to temporarily control bleeding from esophageal varices. It contains 2 balloons and 3 lumens. The gastric lumen drains stomach contents, the esophageal balloon compresses bleeding varices above the esophageal sphincter, and the gastric balloon compresses from below. A weight is attached to the external end of the tube to provide tension and hold the gastric balloon securely in place below the esophageal sphincter. Airway obstruction can occur if the balloon tamponade tube becomes displaced and a balloon migrates into the oropharynx. Scissors are kept at the bedside as a precaution; in the event of airway obstruction, the nurse can emergently cut the tube for rapid balloon deflation and tube removal (Option 2). (Option 1) If airway obstruction occurs, the nurse should first clear the airway and then ensure that the client is stable before contacting the health care provider. (Option 3) Low intermittent suction to the gastric lumen of a balloon tamponade tube is used to drain stomach contents. Increasing the suction would not be indicated if the tube has become displaced. (Option 4) If the balloon tamponade tube is displaced and obstructing the airway, changing the client's position will not help until the client's airway is cleared by removing the tube. Educational objective:A balloon tamponade tube is used to compress bleeding esophageal varices. Tube displacement may result in airway obstruction. The nurse should keep scissors at the bedside so that the tube can be emergently cut and removed if respiratory distress develops due to tube displacement. Additional Information Physiological Adaptation NCSBN Client Need

//A 1-year-old child who goes to day care is recovering from an episode of otitis media. Which intervention is most important for the nurse to recommend to the parents in order to prevent recurrence? 1. Exclusive breastfeeding (2%) 2. Not sending the child to day care (1%) 3. Preventing water from entering the ear (44%) 4. Smoking cessation by the parents (51%) OmittedCorrect answer 4 51%Answered correctly

Otitis media (OM) is the inflammation or infection of the middle ear resulting from dysfunction of the eustachian tube. OM typically occurs in infants and children under age 2, sometimes following a respiratory tract infection. The eustachian tubes in infants and young children are short, straight, and fairly horizontal, which results in ineffective drainage and protection from respiratory secretions. Infants with exposure to tobacco smoke are at risk for OM due to the resulting respiratory inflammation. OM risk is also higher with activities such as using a pacifier or drinking from a bottle when lying down as these allow fluid to pool in the mouth and then reach the eustachian tubes. Key preventive measures include eliminating exposure to smoke, obtaining routine immunizations to prevent infection, and reducing or eliminating use of a pacifier after age 6 months. (Option 1) Breast-fed infants have a decreased risk for OM, possibly due to the semivertical position used when breastfeeding, which reduces reflux to the eustachian tubes. Exclusive breastfeeding is recommended for the first 6 months. However, this client is age 1 and should be receiving a varied, healthy intake of solid food at this time. (Option 2) Day care attendance is a significant risk factor to the development of OM. However, the recommendation to avoid day care is usually not practical as many parents must work outside of the home. (Option 3) Excess water in the ears from bathing or swimming can alter the protective environment of the external ear and contribute to otitis externa, known as swimmer's ear; however, this does not contribute to OM. Educational objective:Otitis media, inflammation of the middle ear, commonly occurs in children under age 2. Key interventions for prevention include avoiding exposure to tobacco smoke, obtaining routine immunizations, and discontinuing use of a pacifier after age 6 months. Additional Information Health Promotion and Maintenance NCSBN Client Need

parent calls the nursing triage line during the evening. The parent says that a 7-year-old was found playing in an area with poison ivy and asks what to do. Which is the most important instruction to give the parent? 1. Apply cool, wet compresses for itching (6%) 2. Apply topical cortisone ointment to the area (6%) 3. Discourage the child from scratching the area (16%) 4. Wash the skin where the contact occurred (71%) OmittedCorrect answer 4 71%Answered correctly

Poison ivy can cause a contact dermatitis rash in those who are sensitive to the oily resin found on the leaves, stems, and roots of the plant. About 50% of people who come in contact with the plant develop a rash. It is often linear in appearance where the plant brushed against the skin. The rash develops 12-48 hours after exposure and can last for several weeks. The severity of the rash depends on the amount of resin on the skin. It is most important to first thoroughly wash the area to remove the resin and prevent its spread to other areas of the body. (Options 1, 2, and 3) Applying cool, wet compresses; applying topical cortisone; and discouraging the child from scratching the area are all appropriate after the rash has developed. Washing the area has the highest priority and is most important immediately after exposure. Educational objective:Immediately after exposure to poison ivy, the client should be instructed to thoroughly wash the area to remove the oily resin, which is responsible for causing the rash that follows in 12-48 hours. Additional Information Physiological Adaptation NCSBN Client Need

The nurse plans to start an IV line to infuse 2 units of packed red blood cells for a stable 42-year-old client with a gastrointestinal bleed. Which IV catheter size is best? 1. 14-gauge (5%) 2. 18-gauge (67%) 3. 20-gauge (19%) 4. 22-gauge (6%) OmittedCorrect answer 2 67%Answered correctly

When selecting catheter size, the need for rapid fluid administration and the type of fluid administered versus client discomfort should be assessed. A lower IV catheter gauge number corresponds to a larger bore IV catheter. A 14-gauge (large-bore) catheter may be used for administering fluids and drugs in an emergency or prehospital setting, or for hypovolemic shock (Option 1). In somewhat stable adult clients who require large amounts of fluids or blood, an 18-gauge catheter is preferred. (Options 3 and 4) A 20-22-gauge catheter is sufficient for administering general IV fluids and medications to adult clients; a 20-gauge is acceptable for blood transfusion. However, 20-22-gauge is not preferred for blood administration. A 24-gauge catheter is recommended for children and some older adults with small, fragile veins. Educational objective:A 14-gauge (large-bore) catheter is used to administer fluids and drugs in a prehospital or emergency setting, or for hypovolemic shock. An 18-gauge catheter is typically indicated for infusing blood or large amounts of fluid in adults.

/There has been a major community disaster. Stable clients need to be discharged to make more beds available for the victims. Which clients could be discharged safely? Select all that apply. 1. Diagnosed with endocarditis on antibiotics with a peripherally inserted central catheter (PICC) line 2. History of multiple sclerosis with ataxia and diplopia 3. One day postoperative from a hemicolectomy 4. Reporting abdominal pain with coffee ground emesis 5. Taking warfarin with prothrombin time/International Normalized Ratio of 2x control value OmittedCorrect answer 1,2,5 18%Answered correctly

Ataxia and diplopia are expected signs/symptoms of multiple sclerosis. Two times the control value demonstrates that warfarin has reached a therapeutic level. The long-term antibiotic course (and follow-up lab work) can continue at home through the PICC line (Options 1, 2, and 5). (Option 3) Large intestine peristalsis does not return for up to 3-5 days. The client cannot be discharged until able to tolerate oral intake with normal elimination. The client has to at least be passing flatus. (Option 4) Coffee ground emesis indicates upper gastrointestinal bleeding. The etiology and treatment need to be determined before the client is discharged. Educational objective:Those who are stable for discharge include the client with multiple sclerosis with ataxia and diplopia, the client on warfarin (Coumadin) that has reached the therapeutic effect, and the client with a PICC line for a long-term antibiotic course. Additional Information Safety and Infection Control NCSBN Client Need

The nurse is caring for a 72-year-old client 1 day postoperative colectomy. The nurse assesses an increased work of breathing, diminished breath sounds at the bases with fine inspiratory crackles, respirations 12/min and shallow, and pulse oximetry 96% on 2 L oxygen. There is no jugular venous distension or peripheral edema. Pain is regulated with client-controlled morphine. Which prescription does the nurse anticipate? 1. Bolus dose of IV morphine (2%) 2. Incentive spirometer (56%) 3. IV furosemide (29%) 4. Non-rebreather mask (11%) OmittedCorrect answer 2 56%Answered correctly

During the initial postoperative period, a client needs respiratory interventions to keep the lungs expanded and prevent atelectasis and postoperative pneumonia. Atelectasis is maximal during the second postoperative night. Clients can be asymptomatic or have increased work of breathing, hypoxia, and basal crackles. Postoperative pain, opioid respiratory depression, limited mobility, and reluctance to take a deep breath due to anticipated pain contribute to postoperative atelectasis. The elderly and postoperative abdominal and thoracic surgery clients are at increased risk for atelectasis. The incentive spirometer encourages the client to breathe deeply with maximum inspiration. This action improves ventilation and oxygenation by expanding the lungs, encourages coughing, and prevents or improves atelectasis. It is the most appropriate prescription for this client. (Option 1) In a client whose pain is regulated with client-controlled analgesia (eg, morphine), administration of a bolus dose is not indicated and may increase the risk for respiratory depression. (Option 3) Fine crackles in the lungs usually indicate atelectasis. The presence of coarse crackles, elevated jugular venous distension, and peripheral edema usually indicates volume overload (fluid in the alveoli). In addition, clients with fluid overload breathe at a rapid rate (tachypnea) rather than take slow, shallow breaths. IV furosemide (Lasix) is an appropriate intervention for volume overload but not for atelectasis. (Option 4) As-needed oxygen may be prescribed postoperatively, especially with blood loss. A non-rebreather mask, which has 100% oxygen, is not indicated in this client as the pulse oximeter shows 96% saturation, indicating adequate oxygenation. Educational objective:The incentive spirometer is a handheld, inexpensive breathing device. It encourages the client to breathe deeply with maximum inspiration, which improves ventilation and oxygenation and encourages coughing. The incentive spirometer is used to prevent or improve atelectasis in clients who are postoperative, have respiratory problems (eg, pneumonia), or have experienced trauma. Additional Information Physiological Adaptation NCSBN Client Need

The nurse has provided teaching about home care to the parent of a 10-year-old with cystic fibrosis. Which of the following statements by the parent indicates that teaching has been effective? Select all that apply. 1. "Chest physiotherapy is administered only if respiratory symptoms worsen." 2. "I will give my child pancreatic enzymes with all meals and snacks." 3. "I will increase my child's salt intake during hot weather." 4. "Our child will need a high-carbohydrate, high-protein diet." 5. "We will limit our child's participation in sports activities." OmittedCorrect answer 2,3,4 21%Answered correctly

In clients with cystic fibrosis (CF), a defective protein responsible for transporting sodium and chloride causes exocrine gland secretions to be thicker and stickier than normal. Viscous respiratory secretions accumulate, resulting in impaired airway clearance and a chronic cough. Clients eventually develop chronic lung disease, which predisposes them to recurrent respiratory infections. Pancreatic enzyme secretion, needed for digestion and absorption of nutrients, is also impaired because thick secretions block pancreatic ducts. Therefore, the client needs supplemental enzymes with all meals and snacks (Option 2). The client also requires multiple vitamins and a diet high in carbohydrates, protein, and fat to help meet nutritional requirements for growth (Option 4). Sweat gland abnormalities prevent sodium and chloride reabsorption, causing increased salt loss, dehydration, and hyponatremia during times of significant perspiration. Therefore, parents should increase the child's salt intake and fluids during hot weather, exercise, or fever (Option 3). (Option 1) Regardless of symptoms, clients should incorporate chest physiotherapy (eg, percussion, vibration, postural drainage) into their daily routine to improve mucus clearance and lung function. (Option 5) The parents should encourage physical activity as tolerated, which helps to thin secretions and remove them from airways and improves muscle strength and lung capacity. Educational objective:Cystic fibrosis causes increased viscosity of exocrine gland secretions. Clients require pancreatic enzyme supplements with meals and snacks; a diet high in carbohydrates, protein, and fat; and increased salt intake during times of significant perspiration. Clients should also incorporate chest physiotherapy and exercise into their daily routine. Additional Information Physiological Adaptation NCSBN Client Need

//During a routine clinic visit, the nurse is providing education to a 24-year-old female client with Marfan syndrome and aortic root dilation. Which statement made by the nurse is appropriate? 1. "Call the health care provider to stop your beta blocker if pregnancy occurs." (21%) 2. "If you plan to become pregnant, it is best to wait a few years and plan it at an older age." (2%) 3. "It is important to consistently use a reliable form of birth control." (69%) 4. "Your condition is not inheritable to your future children." (6%) OmittedCorrect answer 3 69%Answered correctly

Marfan syndrome is a connective tissue disorder that causes visual and cardiac defects and a distinct long, slender body type. In Marfan syndrome with aortic vessel involvement, the root of the aorta is dilated or weakened, increasing the risk of aortic dissection and aortic rupture. Increases in blood volume and cardiac workload that occur during pregnancy may worsen aortic root dilation and further increase the risk of aortic dissection/rupture. Pregnancy in clients with Marfan syndrome, especially those with aortic root dilation, poses a high risk of maternal mortality. Clients should be instructed about the importance of consistently using reliable birth control methods to prevent pregnancy (Option 3). (Option 1) Beta blockers are commonly used to treat clients with Marfan syndrome to limit aortic root dilation. Such medications are generally safe to use during pregnancy, so the client should not discontinue therapy unless directed to do so by the health care provider. (Option 2) Clients with Marfan syndrome considering pregnancy should be counseled to complete childbearing in early adulthood because aortic root dilation and the risk of aortic dissection/rupture increase with time. (Option 4) Marfan syndrome is an autosomal dominant condition with a 50% chance of inheritance in offspring. Educational objective:Marfan syndrome affects the connective tissues and is associated with dilation of the aortic root. Clients with Marfan syndrome are at high risk of mortality during pregnancy due to the potential for aortic dissection. Consistent use of reliable birth control is essential for preventing pregnancy.

The nurse is caring for a client in the medical-surgical unit who has delirium according to the Confusion Assessment Method assessment tool. Which of the following assessment findings are likely contributing to the client's delirium? Select all that apply. 1. Multiple doses of IV hydromorphone administered in the past 12 hours 2. Serum sodium of 123 mEq/L (123 mmol/L) 3. SpO2 of 82% on room air 4. Temperature of 103.1 F (39.5 C) 5. Urine culture positive for gram-positive cocci in chains OmittedCorrect answer 1,2,3,4,5 27%Answered correctly

NCLEX® CHANGE AS OF 2017 - Please note that select-all-that-apply (SATA) questions on NCLEX can now include any number of correct responses. Only ONE option or up to ALL options may be correct. UWorld questions now reflect this change. Visit NCSBN® NCLEX FAQs for more information. Delirium is characterized by an acute or fluctuating change in mental status that is often reversible and related to an underlying medical condition. Evidence-based assessment tools, such as the Confusion Assessment Method (CAM), help clinicians quickly recognize delirium. Criteria of the CAM tool include an acute or fluctuating change in mentation, inattention, disorganized thinking, and altered level of consciousness. Precipitating factors of delirium are numerous and include: Medications (eg, opioids, anticholinergics) (Option 1) Electrolyte imbalances (eg, hyponatremia) (Option 2) Hypoxia (Option 3) Acute infection (eg, fever, positive culture) (Options 4 and 5) Sleep deprivation Dehydration or malnutrition Metabolic disorders (eg, hypoglycemia) Nursing interventions include treating the underlying cause as prescribed to resolve delirium (eg, antibiotics, supplemental oxygen), maintaining a safe environment (eg, continuous monitoring, room near the nurses' station, bed alarm), reorienting the client frequently, promoting a regular sleep cycle, providing familiar items from home, and encouraging family and friends to stay with the client. Educational objective:The Confusion Assessment Method is an assessment tool that helps clinicians quickly recognize delirium, which is characterized by an acute or fluctuating change in mental status and is often caused by medications (eg, opioids) or an underlying medical condition (eg, hypoxia, electrolyte imbalances, infection).

Progress notes 1845Productive cough of large amount of tan mucus, scattered rhonchi, and crackles in bases. Arterial blood gas (ABG) results: PaCO2 35 mm Hg, PaO2 90 mm Hg on nasal oxygen at 6L/min. Temperature 101.1 F (38.3 C). On vancomycin for 2 days.______________RN 1945 Repeat ABG: PaCO2 33 mm Hg, PaO2 89 mm Hg on 50% oxygen via face mask.______________RN 2045 Repeat ABG: PaCO2 32 mm Hg, PaO2 86 mm Hg on 100% oxygen via total rebreather mask.______________RN A client with pneumonia is transferred from the medical unit to the intensive care unit due to sepsis and worsening respiratory failure. Based on the nurse's progress note, which assessment data are most important for the nurse to report to the health care provider (HCP)? Click on the exhibit button for additional information. 1. Cough with mucus production (1%) 2. Refractory hypoxemia (82%) 3. Scattered rhonchi and crackles (9%) 4. Temperature 101 F (38.3 C) (6%) OmittedCorrect answer 2 82%Answered correctly

Refractory hypoxemia is the hallmark of acute respiratory distress syndrome (ARDS), a progressive form of acute respiratory failure that has a high mortality rate. It can develop following a pulmonary insult (eg, aspiration, pneumonia, toxic inhalation) or nonpulmonary insult (eg, sepsis, multiple blood transfusions, trauma) to the lung. The insult triggers a massive inflammatory response that causes the lung tissue to release inflammatory mediators (leukotrienes, proteases) that cause damage to the alveolar-capillary (A-C) membrane. As a result of the damage, the A-C membrane becomes more permeable, and intravascular fluid then leaks into the alveolar space, resulting in a noncardiogenic pulmonary edema. The lungs become stiff and noncompliant, which makes ventilation and oxygenation less than optimal and results in increased work of breathing, tachypnea and alkalosis, atelectasis, and refractory hypoxemia. Profound hypoxemia despite high concentrations of oxygen is a key sign of ARDS and is the most important assessment finding to report to the HCP. (Options 1 and 3) Cough with mucus production and scattered rhonchi and crackles are expected findings in a client with pneumonia. (Option 4) Temperature is an expected finding in a client with pneumonia who is receiving antibiotic therapy. The white blood cell count can still be elevated after 2 days of antibiotic therapy. Educational objective:Refractory hypoxemia is the inability to improve oxygenation with increases in oxygen concentration. It is the hallmark of ARDS, a progressive form of acute respiratory failure that has a high mortality rate.

The emergency department nurse prepares a male client for surgery. The client was admitted with a traumatic open fracture of the femur, hematocrit of 36% (0.36), and hemoglobin of 12 g/dL (120 g/L). Which prescription should the nurse validate with the health care provider before administration? 1. Cefazolin (8%) 2. Enoxaparin (75%) 3. Morphine (4%) 4. Tetanus toxoid (10%) OmittedCorrect answer 2 75%Answered correctly

The Joint Commission Surgical Improvement Project CORE measure set has shown that preventives (eg, heparin, enoxaparin, aspirin) in select surgical procedures, given 24 hours before and after surgery, reduce the risk of venous thromboembolism. However, the estimated blood loss in a client with a fracture can be significant depending on the site (eg, 250-1200 mL). Although this client's admission hematocrit (36% [0.36]) and hemoglobin (12 g/dL [120 g/L]) are only slightly low for an adult male (normal: 39%-50% [0.39-0.50], 13.2-17.3 g/dL [132-173 g/L]), the blood loss may not yet be evident. Therefore, the nurse would validate the prescription for enoxaparin (Lovenox) with the health care provider before administration. Medications commonly prescribed for a client with an open fracture include: Cefazolin (Ancef), a bone-penetrating cephalosporin antibiotic that is active against skin flora (Staphylococcus aureus); it is given prophylactically before and after surgery to prevent infection (Option 1) Cyclobenzaprine (Flexeril), a central and peripheral muscle relaxant given to treat pain associated with muscle spasm; carisoprodol (Soma) or methocarbamol (Robaxin) can also be prescribed Tetanus and diphtheria toxoid, an immunization given prophylactically to prevent infection (Clostridium tetani) if immunizations are not up to date (>10 years), unavailable, or unknown (Option 4) Ketorolac (Toradol), a nonsteroidal anti-inflammatory drug given to decrease inflammation and pain Opioids (eg, morphine, hydrocodone [Vicodin]), given for analgesia (Option 3) Educational objective:Medications commonly prescribed for a client with an open fracture to prevent infection and treat pain and muscle spasm include cefazolin (Ancef), tetanus toxoid, ketorolac (Toradol), opioids, and cyclobenzaprine (Flexeril).

//Which statement made by the client demonstrates a correct understanding of the home care of an ascending colostomy? 1. "I will avoid eating foods such as broccoli and cauliflower." (42%) 2. "I will empty the pouch when it is one-half full of stool." (36%) 3. "I will irrigate the colostomy to promote regular bowel movements." (16%) 4. "I will restrict my fluid intake to 2,000 milliliters of fluid a day." (4%) OmittedCorrect answer 1 42%Answered correctly

A colostomy is a surgical procedure that creates an opening (stoma) in the abdominal wall for the passage of stool to bypass an obstructed or diseased portion of the colon. Stool drains through the intestinal stoma into a pouch device secured to the skin. Colostomies can be performed on any part of the colon (ascending, transverse, descending, and sigmoid). Depending on the location of the colostomy, characteristics of the stool will vary, with the stool becoming more solid as it passes through the colon. Proper care of the stoma and pouch appliance that should be taught to the client or caregiver includes the following: Ensure sufficient fluid intake (at least 3,000 mL/day unless contraindicated) to prevent dehydration; identify times to increase fluid requirements (hot weather, increased perspiration, diarrhea) (Option 4). Identify and eliminate foods that cause gas and odor (broccoli, cauliflower, dried beans, brussels sprouts) (Option 1). Empty the pouch when it becomes one-third full to prevent leaks due to increasing pouch weight (Option 2). (Option 3) Stool produced in the ascending and transverse colon is semiliquid, which eliminates the need for irrigation. Irrigation to promote a bowel regimen may be useful for descending or sigmoid colostomies as the stool is more formed. Educational objective:The stool changes from liquid to more solid as it passes through the colon. Proper care of the ostomy and pouching device in clients with a colostomy includes ensuring sufficient fluid intake, preventing gas and odor, and changing the pouching system when it becomes one-third full to prevent leaks.

While preparing to insert a peripheral IV line, the nurse notices scarring near the client's left axilla. The client confirms a history of left breast cancer and modified radical mastectomy. Which actions should the nurse take? Select all that apply. 1. Advance the entire stylet into the vein upon venipuncture 2. Insert the IV line into the most distal site of the right arm 3. Place an appropriate precaution sign above the bed 4. Review the medical record for history of mastectomy 5. Teach the client to keep the left arm in a dependent position OmittedCorrect answer 2,3,4 33%Answered correctly

A modified radical mastectomy includes removal of axillary lymph nodes that are involved in lymphatic drainage of the arm. Any trauma (eg, IV extravasation) to the arm on the operative side can result in lymphedema, characterized by painful and lengthy swelling, as normal lymphatic circulation is impaired by scarring. Therefore, starting an IV line in this arm is contraindicated. The nurse should insert the IV line into the most distal site of the unaffected side (Option 2). For client safety, it is also important to ensure documentation of the mastectomy history, place a restricted extremity armband on the affected arm, and place a sign above the client's bed notifying hospital staff of necessary mastectomy precautions (eg, no blood pressure measurements, venipuncture, or IV lines) (Options 3 and 4). In general, venipuncture is contraindicated in upper extremities affected by: Weakness Paralysis Infection Arteriovenous fistula or graft (used for hemodialysis) Impaired lymphatic drainage (prior mastectomy) (Option 1) The stylet should be advanced until blood return is seen (approximately ¼ inch). If advanced fully, the stylet may penetrate the posterior wall of the vein and cause a hematoma. (Option 5) Keeping the affected arm in a dependent position for a long time can increase lymphedema. The client should be reminded that raising the limb helps drainage. Educational objective:IV line insertion is contraindicated on the operative side of clients with a prior mastectomy. Additional contraindications for IV line insertion include weakness, paralysis, or infection of the arm; or presence of an arteriovenous fistula. Additional Information Pharmacological and Parenteral Therapies NCSBN Client Need

The nurse develops a care plan for a critically ill client with acute respiratory distress syndrome (ARDS) who is on a mechanical ventilator. What is the priority nursing diagnosis (ND)? 1. Imbalanced nutrition (1%) 2. Impaired gas exchange (82%) 3. Impaired tissue integrity (2%) 4. Risk for infection (13%) OmittedCorrect answer 2 82%Answered correctly

ARDS involves damage to the alveolar-capillary membrane, the blood-gas barrier across which oxygen diffuses into the alveoli. When the membrane is damaged, the alveoli collapse and fluid leaks into the alveolar space and impairs gas exchange. Impaired gas exchange related to alveolar-capillary changes and ventilation-perfusion imbalance is the priority ND for a client with ARDS. (Option 1) Imbalanced nutrition (less than body requirements) related to increased metabolic needs and inability to ingest foods due to endotracheal intubation, is an appropriate ND for the client with ARDS. However, it does not pose the greatest threat to survival and is not the priority ND. (Option 3) Impaired tissue (integumentary) related to altered circulation, immobility, and nutritional deficits is an appropriate ND for the client with ARDS. However, it does not pose the greatest threat to survival and is not the priority ND. (Option 4) Risk for infection related to the presence of an endotracheal tube, frequent suctioning, intravenous devices, and indwelling catheters is an appropriate ND for the client with ARDS. However, it does not pose the greatest threat to survival and is not the priority ND. Educational objective:ARDS involves damage to the alveolar-capillary membrane, resulting in fluid leakage into the alveolar space. Impaired gas exchange related to alveolar-capillary changes and ventilation-perfusion imbalance is an appropriate ND for a client with ARDS. Additional Information Physiological Adaptation NCSBN Client Need

/During a screening clinic, the nurse performs a health assessment on several adult clients. Which finding by the nurse is most important to report to the primary health care provider? 1. Body mass index (BMI) of 23 kg/m2 (2%) 2. Brownish skin thickening on the neck (67%) 3. Fasting total cholesterol of 180 mg/dL (4.7 mmol/L) (10%) 4. Round 3x3 mm pale pink mole (19%) OmittedCorrect answer 2 67%Answered correctl

Acanthosis nigricans is a skin disorder characterized by the presence of symmetric, hyperpigmented velvety plaques located in flexural and intertriginous regions of skin (axilla, neck). Skin tags (acrochordons) are commonly present on regions affected by acanthosis nigricans. Both indicate insulin resistance (diabetic dermopathy). The client should be referred to the primary health care provider for evaluation of undiagnosed diabetes mellitus and/or metabolic syndrome. (Option 1) A BMI of 18.5 to 24.9 kg/m2 indicates a normal weight. (Option 3) A fasting total cholesterol <200 mg/dL (5.2 mmol/L) is normal. (Option 4) Although any change or growth of a mole should be reported, a pale or brown round mole <5 mm is typically a normal finding. Educational objective:Acanthosis nigricans is a velvety light brownish to black skin thickening seen in the axillae, neck, or flexures and is indicative of insulin resistance (diabetic dermopathy). Skin tags (acrochordons) are commonly present on regions affected by acanthosis nigricans.

//The nurse assesses a female client with a diagnosis of primary adrenal insufficiency (Addison disease). The nurse recognizes which finding associated with the disease? 1. Bronze pigmentation of skin (75%) 2. Increased body or facial hair (10%) 3. Purple or red striae on the abdomen (8%) 4. Supraclavicular fat pad (5%) OmittedCorrect answer 1 75%Answered correctly

Addison disease, or primary adrenocortical insufficiency, is also described as hypofunction of the adrenal cortex. The adrenal gland is responsible for secretion of glucocorticoids, androgens, and mineralocorticoids. Bronze hyperpigmentation of the skin in sun-exposed areas is caused by an increase in adrenocorticotropic hormone (ACTH) by the pituitary in response to low cortisol (ie, glucocorticoid) levels (Option 1). Clients with Addison disease may also have vitiligo, or patchy/blotchy skin, which is usually present when the etiology of the disease is an autoimmune problem. The immune cells are thought to destroy melanocytes which produce melanin (or brown pigment), resulting in a patchy appearance. Other common manifestations of Addison disease include the following: Slow, progressive onset of weakness and fatigue Anorexia and weight loss Orthostatic hypotension Hyponatremia and hyperkalemia Salt cravings Nausea and vomiting Depression and irritability (Options 2, 3, and 4) Purple striae, hirsutism (increased facial and body hair), and a supraclavicular fat pad (ie, buffalo hump) are characteristics of Cushing syndrome, a condition associated with excess corticosteroid production. In contrast, Addison disease is a condition of hyposecretion of glucocorticoids. Educational objective:Hyperpigmentation of the skin is a common characteristic of Addison disease, or primary adrenocortical insufficiency, which can also cause hypotension, hyponatremia, hyperkalemia, and vitiligo.

An elderly client visits the clinic for an annual examination, which includes updating the client's advance care plan. When assessing the client's advance care planning needs, which topics should the nurse discuss? Select all that apply. 1. Financial power of attorney 2. Health care proxy 3. Life insurance beneficiary 4. Living will 5. Safe deposit box OmittedCorrect answer 2,4 58%Answered correctly

Advance care planning is an ongoing process that should be revisited yearly and after changes in condition. Legal documentation is needed to ensure that the client's advance care plan is carried out correctly. Advance care planning documents may include the following: A health care proxy (durable power of attorney for health care or medical power of attorney) is a person appointed by the client to make decisions on behalf of the client. The proxy document only goes into effect when the health care team determines that the client lacks the capacity to make decisions. This should be deactivated if the client regains decision-making capacity. A living will is an advanced directive describing the type of life-sustaining treatments (eg, cardiopulmonary resuscitation, intubation, mechanical ventilation, feeding tube) that the client wants initiated if unable to make decisions. (Option 1) The financial power of attorney form can help clients having difficulty managing financial affairs and needing someone to help; however, it is not part of the advance care planning process. (Option 3) The client must choose a beneficiary for life insurance policies; however, life insurance is not part of the advance care plan. (Option 5) A safe deposit box can be a good place to ensure that legal documents are stored safely. It is not part of the advance care planning process. Educational objective:Advance care planning allows the client to determine desired treatments (eg, cardiopulmonary resuscitation, intubation, mechanical ventilation) and decision makers in the event the client is unable to do so. Advanced directives are legal documents outlining these wishes and include living wills and health care proxies (durable powers of attorney for health care or medical power or attorney).

A client with an asthma exacerbation has been using her albuterol rescue inhaler 10-12 times a day because she cannot take a full breath. What possible side effects of albuterol does the nurse anticipate the client will report? Select all that apply. 1. Constipation 2. Difficulty sleeping 3. Hives with pruritus 4. Palpitations 5. Tremor OmittedCorrect answer 2,4,5 51%Answered correctly

Albuterol is a short-term beta-adrenergic agonist used as a rescue inhaler to treat reversible airway obstruction associated with asthma. Dosing in an acute asthma exacerbation should not exceed 2-4 puffs every 20 minutes x 3. If albuterol is not effective, an inhaled corticosteroid is indicated to treat the inflammatory component of the disease. Albuterol is a sympathomimetic drug. Expected side effects mimic manifestations related to stimulation of the sympathetic nervous system, and commonly include insomnia, nausea and vomiting, palpitations (from tachycardia), and mild tremor. (Option 1) Constipation is not a common side effect of inhaled beta-agonist drugs. (Option 3) Hives can occur as a sign of an allergic reaction and are not a common anticipated side effect of an inhaled beta-agonist drug. Educational objective:Albuterol is a short-term beta-agonist rescue drug used to control symptoms of airway obstruction and promote bronchodilation. It is a sympathomimetic drug; common expected side effects include insomnia, nausea and vomiting, palpitations (tachycardia), and mild tremor. Additional Information Pharmacological and Parenteral Therapies NCSBN Client Need

A client suffering from chronic kidney disease is scheduled to receive recombinant human erythropoietin and iron sucrose. The client's hemoglobin is 9.7 g/dL (97 g/L) and hematocrit is 29% (0.29). What is the appropriate nursing action? 1. Administer the erythropoietin in the client's ventrogluteal muscle (42%) 2. Check blood pressure prior to administering the erythropoietin (39%) 3. Hold the client's next scheduled iron sucrose dose (3%) 4. Hold the erythropoietin and inform the health care provider (15%) OmittedCorrect answer 2 39%Answered correctly

Anemia associated with chronic kidney disease is treated with recombinant human erythropoietin (Epogen/Procrit, epoetin). Therapy is initiated when hemoglobin is <10 g/dL (100 g/L) to alleviate the symptoms of anemia (eg, fatigue) and the need for blood transfusions. Therapy should be discontinued or the dose reduced for hemoglobin >11 g/dL (110 g/L) to prevent venous thromboembolism and adverse cardiovascular outcomes from blood thickened by high concentrations of RBCs. Hypertension is a major adverse effect of erythropoietin administration. Therefore, uncontrolled hypertension is a contraindication to recombinant erythropoietin therapy. Blood pressure should be well controlled prior to administering erythropoietin (Option 2). (Option 1) Erythropoietin is administered intravenously or in any subcutaneous area (not intramuscularly). (Option 3) Iron in the form of iron sucrose (Venofer) or ferric gluconate (Ferrlecit) may be prescribed to promote an adequate response to erythropoietin. Adequate stores of iron, vitamin B12, and folic acid are required for the erythropoietin to work. (Option 4) The dose should be held if the client has a hemoglobin level >11 g/dL (110 g/L) or uncontrolled hypertension. Educational objective:Anemia of chronic kidney disease is treated with recombinant human erythropoietin for hemoglobin <10 g/dL (100 g/L). Hemoglobin levels >11 g/dL (110 g/L) are associated with thromboembolic and cardiovascular events. Uncontrolled hypertension is a contraindication to recombinant human erythropoietin therapy. Additional Information Pharmacological and Parenteral Therapies NCSBN Client Need

/The charge nurse must assign a semi-private room to a client with diabetes mellitus admitted for IV antibiotic therapy to treat leg cellulitis. Which of the 4 room assignments is the best option for this client? 1. Room 1: Client 1 day postoperative laparoscopic cholecystectomy who is awaiting discharge (31%) 2. Room 2: Client with dementia and urinary incontinence wearing an external urine collection device (26%) 3. Room 3: Client with history of splenectomy 15 years ago, now admitted for pulmonary embolism (19%) 4. Room 4: Client with lupus nephritis who is prescribed treatment with azathioprine (21%) OmittedCorrect answer 2 26%Answered correctly

Cellulitis is a common skin bacterial infection that is usually treated with IV antibiotics in clients with diabetes mellitus. Room 2 is the best assignment option for this client with cellulitis. The client with dementia and urinary incontinence who has an external urinary condom catheter is the least susceptible to infection compared to those in rooms 1, 3, and 4. (Option 1) The client who is 1 day postoperative laparoscopic cholecystectomy (surgical procedure with small incisions) is at increased risk for infection. The client with cellulitis should not be placed in room 1. (Option 3) Although this client has pulmonary embolism, the history of prior splenectomy leads to a very high lifelong risk of rapid sepsis. Splenectomy clients need vaccination against encapsulated organisms (eg, pneumococcus, meningococcus, and Haemophilus influenzae type B). Even a low-grade fever should be taken seriously in these clients. The client with cellulitis should not be placed in room 3. (Option 4) Lupus nephritis is a serious renal complication of systemic lupus erythematosus (SLE), an inflammatory autoimmune disease that can lead to end-stage kidney disease. The systemic disease and the immunosuppressant (azathioprine [Imuran]) prescribed to slow its progression increase infection risk. The client with cellulitis should not be placed in room 4. Educational objective:A client with an infection should not be assigned to a semi-private room with a client who had surgery or is immunocompromised and receiving immunosuppressants as these clients are highly susceptible to infection. Post-splenectomy clients are also at lifelong risk for rapid sepsis. Additional Information Safety and Infection Control NCSBN Client Need

/The nurse is caring for the assigned clients on a pediatric inpatient unit. Which client is the priority? 1. 8-year-old with sickle cell crisis who has sudden-onset unilateral arm weakness (77%) 2. 11-year-old with viral meningitis requesting pain medication for headache (10%) 3. Male child scheduled for surgery for intussusception who has reddish mucoid stool (6%) 4. Male child with hemophilia who has hemarthrosis and is receiving desmopressin (5%) OmittedCorrect answer 1 77%Answered correctly

Children can have strokes. Ischemic strokes are more common in children with sickle cell disease. Other causes can include carotid abnormalities/dissection. The most common presentation of an ischemic stroke is the sudden onset of numbness or weakness of an arm and/or leg. These are handled with a similar emergent approach as for stroke in an adult. Children may require exchange blood transfusion to prevent the stroke from worsening. (Option 2) Viral meningitis can cause fever, headache, and meningeal signs (neck stiffness). Headache is expected and not a priority over a client with stroke. (Option 3) Intussusception occurs when one portion of the intestine prolapses and then telescopes into another. It is a frequent cause of intestinal obstruction during infancy. Onset is abrupt, initially with pain and brown stool. The condition then progresses to bilious emesis, palpable abdominal mass, and stools with a red, "currant jelly" appearance due to blood and mucus. This is an expected finding for this condition, and surgery is already scheduled to address it. (Option 4) Hemophilia is seen primarily in males and is due to a lack of clotting factors. Symptoms include spontaneous bleeding (hemarthrosis) into the joints, especially the knee, ankle, or elbow. Treatment includes replacing the missing clotting factor. Desmopressin (DDAVP) stimulates the release of factor VIII. The child is receiving treatment already and joint rest has been prescribed. The sudden neurological change in the child with sickle cell crisis is a priority. Educational objective:Children can have strokes. These usually are caused by clotting or vascular issues and require similar emergent care as adults. Desmopressin (DDAVP) is used to treat hemophilia.

A client with mitral valve prolapse (MVP) has been experiencing occasional palpitations, lightheadedness, and dizziness. The health care provider prescribes a beta blocker. What additional teaching should the nurse include for this client? 1. Avoid aerobic exercise (7%) 2. Ensure you receive antibiotics prior to dental work (26%) 3. Stay well hydrated and avoid caffeine (50%) 4. Wear a medical alert bracelet (14%) OmittedCorrect answer 3 50%Answered correctly

Clients with MVP may have palpitations, dizziness, and lightheadedness. Chest pain can occur but its etiology is unknown in this client population. It may be a result of abnormal tension on the papillary muscles. Chest pain that occurs in MVP does not typically respond to antianginal treatment such as nitrates. Beta blockers may be prescribed for palpitations and chest pain. Client teaching for MVP includes the following: Adopt healthy eating habits and avoid caffeine as it is a stimulant and may exacerbate symptoms (Option 3) Check ingredients of over-the-counter medications or diet pills for stimulants such as caffeine or ephedrine as they can exacerbate symptoms Reduce stress and avoid alcohol use (Option 1) Clients should be taught to begin or maintain an exercise program, preferably aerobic exercise, to achieve optimal health. (Option 2) Although MVP may place the client at an increased risk for infective endocarditis, there is no clinical evidence to support the need for prophylactic antibiotics prior to dental procedures. Antibiotic prophylaxis is indicated for clients who have prosthetic valve replacement, repaired valves, or a history of infectious endocarditis. (Option 4) There is no need for a medical alert bracelet. MVP is usually a benign condition. Educational objective:The nurse should teach the client with MVP to stay hydrated, avoid caffeine and alcohol, exercise regularly, reduce stress, and take beta blockers as prescribed for palpitations and chest pain. Nitrates are usually not effective for chest pain from MVP.

/A client is hospitalized with worsening chronic heart failure. Which clinical manifestations does the admitting nurse most likely assess in this client? Select all that apply. 1. Crackles on auscultation 2. Dry mucous membranes 3. Increased jugular venous distention 4. Rhonchi on auscultation 5. Skin "tenting" 6. 3+ pitting edema of the lower extremities OmittedCorrect answer 1,3,6 60%Answered correctly

Clients with a diagnosis of chronic congestive heart failure experience clinical manifestations of both right-sided (systemic venous congestion) and left-sided (pulmonary congestion) failure. Crackles are discontinuous, adventitious lung sounds usually heard on inspiration and indicate the presence of pulmonary congestion (left-sided failure) in this client. Increased jugular venous distention reflects an increase in pressure and volume in the systemic circulation, resulting in elevated central venous pressure (CVP) (right-sided failure) in this client. Although dependent pitting edema of the extremities can be associated with other conditions (eg, hypoproteinemia, venous insufficiency), it is related to sodium and fluid retention (right-sided failure) in this client. (Option 2) Dry mucous membranes are associated with dehydration (increased serum sodium level), not fluid overload (heart failure). (Option 4) Rhonchi are continuous lung sounds usually heard on expiration that indicate the presence of secretions in the larger airways. They are not a classic manifestation of chronic heart failure. (Option 5) Poor skin turgor or "tenting" is associated with skin moisture and elasticity. It is usually associated with dehydration, not fluid overload. Educational objective:Clients with chronic heart failure experience clinical manifestations of both right-sided and left-sided failure. Therefore, the nurse must be able to assess for the clinical manifestations related to systemic volume increases and pulmonary congestion.

The nurse is preparing to administer IV cefazolin to a client with cellulitis. The client's allergies are listed as amoxicillin, ciprofloxacin, and sulfa drugs. What should the nurse do first? 1. Administer the medication as prescribed (14%) 2. Clarify the prescription with the health care provider (49%) 3. Inquire about the type of allergic reaction (32%) 4. Notify the pharmacy that the drug is inappropriate (4%) OmittedCorrect answer 3 32%Answered correctly

Clients with an allergy to penicillin antibiotics (eg, amoxicillin, ampicillin) can possibly experience a cross-sensitivity reaction to cephalosporin antibiotics (eg, cefazolin, cephalexin, ceftriaxone), because the drug molecules are structurally similar. The nurse should obtain more information about this client's reported allergies, as reactions range from mild to severe. In particular, the nurse must first assess the type of reaction the client had to amoxicillin (Option 3). The nurse should then clarify the prescription with the health care provider (HCP) prior to administration. If this client's reaction to amoxicillin was a rash or other mild reaction that was not life-threatening, the HCP may decide that cephalosporin can be safely administered. However, cephalosporins are contraindicated for a client with a history of anaphylactic reactions to penicillin, and a different antibiotic should be prescribed. (Option 1) The nurse should hold the medication until more is known about the client's reaction to amoxicillin. (Option 2) The nurse must first obtain more information about the reaction so the HCP can make an informed decision about whether the cephalosporin antibiotic can be administered or should be changed. (Option 4) The nurse must first obtain more information about the type of allergic reaction before notifying the pharmacy. Educational objective:A client with a penicillin allergy may experience a cross-sensitivity reaction to cephalosporin antibiotics (eg, cefazolin, cephalexin, ceftriaxone). Cephalosporins can typically be administered safely to clients with a history of mild allergic reaction (eg, rash), but are contraindicated in clients with a history of penicillin anaphylaxis.

The nurse reviews the medication administration records and laboratory results for assigned clients. Which medication requires that the health care provider be notified before administration? 1. Calcium acetate for a client with a phosphate level of 8.5 mg/dL (2.75 mmol/L) (11%) 2. Clopidogrel for a client with a platelet count of 70,000/mm3 (70 × 109/L) (72%) 3. Magnesium sulfate for a client with a magnesium level of 1.0 mEq/L (0.5 mmol/L) (8%) 4. Metformin for a client with a glycosylated hemoglobin level of 11% (7%) OmittedCorrect answer 2 72%Answered correctly

Clopidogrel (Plavix) is a platelet aggregation inhibitor used to prevent blood clot formation in clients with recent myocardial infarction, acute coronary syndrome, cardiac stents, stroke, or peripheral vascular disease. Because it can cause thrombocytopenia and increase the risk for bleeding, the nurse should notify the health care provider (HCP) of the low platelet count (normal: 150,000-400,000/mm3 [150-400 × 109/L]) before administering clopidogrel. (Option 1) Calcium acetate (PhosLo) is used to control hyperphosphatemia in clients with end-stage kidney disease by binding to phosphate in the intestines and excreting it in the stool. Because the phosphate level is high (normal adult: 2.4-4.4 mg/dL [0.78-1.42 mmol/L]), it is not necessary to notify the HCP. (Option 3) Magnesium sulfate is used to correct hypomagnesemia and treat torsades de pointes and seizures associated with eclampsia. Because the magnesium level is low (normal adult: 1.5-2.5 mEq/L [0.75-1.25 mmol/L]), it is not necessary to notify the HCP. (Option 4) Metformin (Glucophage) is a first-line drug for the control of blood sugar in clients with type 2 diabetes mellitus. Glycosylated hemoglobin (A1C) measures the total hemoglobin that has glucose attached to it, expressed as a percentage. Glucose remains attached to the red blood cell for the life of the cell (about 120 days) and reflects glycemic control over an extended period. The recommended A1C level for a client with diabetes is <7%. Although the A1C level is elevated, the medication would be administered regardless of the result (unless the client is hypoglycemic), so it is not necessary to notify the HCP. Educational objective:Clopidogrel (Plavix) can cause thrombocytopenia (platelet count <150,000/mm3 [150 × 109/L]) and increase a client's risk for bleeding.

The clinic nurse speaks with the spouse of a client being treated for alcohol use disorder. Which statements by the spouse indicate codependence? Select all that apply. 1. "I am focusing on my new hobby and my friends in the book club." 2. "I left and didn't awaken my spouse, who went back to sleep after turning off the alarm clock." 3. "I try to get up early and keep the children from being too loud in the mornings." 4. "If I didn't get so stressed about my job, my spouse wouldn't drink so much." 5. "When my spouse was sick, I called and rescheduled clients so my spouse could rest." OmittedCorrect answer 3,4,5 27%Answered correctly

Codependent behaviors are those that allow the codependent person to maintain control by fulfilling the needs of the addict first. Behaviors such as keeping the addiction secret, suffering physical or psychological abuse from the addict, not allowing the addict to suffer the consequences of actions, and making excuses for the addict's habit are hallmarks of codependency. If the addict isn't happy, the codependent person will try to make the addict happy. Codependent persons will focus all their attention on others at the expense of their own sense of self. Codependent spouses, friends, and family members keep the client from focusing on treatment; this behavior is counterproductive to both themselves and the client. (Option 1) This statement does not represent codependency but rather indicates that the spouse is focusing on the spouse's own growth and needs rather than the needs of others. (Option 2) This statement does not represent codependency; it indicates that the spouse is allowing the client to suffer the consequence of actions. Educational objective:Codependent spouses, friends, and family members can impede treatment progress of clients with substance use disorders. Codependent behaviors include making excuses for a client's drug/alcohol use, putting a client's needs before one's own, and not allowing a client to suffer the consequences of actions.

A registered nurse is precepting a new nurse in the intensive care unit. The client is sedated with propofol, on a mechanical ventilator, and is receiving enteral feeding via nasogastric tube. The new nurse performs interventions to prevent aspiration. The preceptor should intervene if the new nurse performs which of the following actions? 1. Assesses gastric residual volumes every 4 hours (10%) 2. Measures the number of centimeters the feeding tube is secured at the nare every 4 hours (8%) 3. Requests that the physician change the client from continual to bolus feedings (67%) 4. Uses a sedation scale to titrate down the sedation (if possible) (14%) OmittedCorrect answer 3 67%Answered correctly

Critically ill clients are at increased risk for aspiration of oropharyngeal secretions and gastric content. It is common in clients who are intubated, sedated, on a mechanical ventilator, and receiving enteral feedings. The nurse must provide nursing interventions to prevent aspiration and monitor for its signs and symptoms. Clients are at increased risk when receiving bolus rather than continual enteral feedings. Bolus feedings should be avoided in critically ill clients, who are already at increased risk for aspiration. (Option 1) Assessing gastric residual volumes according to institution policy (at least every 4 hours) is standard for clients receiving continual enteral feedings. Increased volumes may indicate poor absorption and increase the risk of regurgitation and aspiration. (Option 2) Measuring the number of centimeters at the nare every 4 hours can help determine if the tube has moved, but it can increase aspiration risk. X-ray confirmation may be necessary if the tube has moved. (Option 4) A sedation scale such as the Ramsay Scale is used to assess level of sedation. It is preferable to keep the client minimally sedated (asleep but arousable). This helps decrease the risk of aspiration. Educational objective:Assessing gastric residual volumes and level of sedation at regular intervals, checking enteral feeding tube placement, and administering continual rather than bolus tube feeding are interventions that help prevent aspiration in critically ill high-risk clients.

The nurse is reviewing new prescriptions from the health care provider. Which prescription would require further clarification? 1. Atorvastatin for hyperlipidemia in a client with angina pectoris (9%) 2. Bupropion for smoking cessation in a client with emphysema (15%) 3. Cyclobenzaprine for muscle spasms in a client with hepatitis (54%) 4. Metronidazole for trichomoniasis in a client with Crohn disease (20%) OmittedCorrect answer 3 54%Answered correctly

Cyclobenzaprine (Flexeril) is a common, centrally acting skeletal muscle relaxant prescribed for muscle spasticity, muscle rigidity, and acute or chronic muscle pain/injury. Centrally acting muscle relaxants interfere with reflexes within the central nervous system (CNS) to decrease muscle spasm and rigidity. Like many medications, muscle relaxants are metabolized by the liver. The presence of liver disease (eg, hepatitis) decreases hepatic metabolism and can cause a buildup of medication, leading to medication toxicity and increased CNS depression (eg, weakness, confusion, drowsiness, lethargy). The prescription for a muscle relaxant would need to be clarified in a client with liver disease (Option 3). (Option 1) Atorvastatin (Lipitor) is a statin prescribed for hyperlipidemia. It is used for primary and secondary prevention of cardiovascular disease and would not warrant further clarification when used in a client with angina pectoris. (Option 2) Bupropion (Wellbutrin, Zyban) and varenicline (Chantix, Champix) are commonly prescribed for smoking cessation. Both bupropion and varenicline can cause serious neuropsychiatric effects (eg, depression, suicide); however, there is no contraindication for clients with emphysema. (Option 4) Metronidazole (Flagyl) is an antibiotic that can be used to treat a Trichomonas infection. There is no contraindication for its use in clients with Crohn disease. Educational objective:Like many medications, skeletal muscle relaxants (eg, cyclobenzaprine) are metabolized hepatically. In the presence of hepatic impairment (eg, hepatitis), drug metabolism is reduced and results in the accumulation of medication in the body, which leads to toxicity and serious adverse effects.

/Yesterday, the client was weaned from the mechanical ventilator and an intravenous infusion of lorazepam. The client has been alert and oriented for 24 hours but is now experiencing confusion. The nurse now evaluates new-onset confusion by assessing the client's sense of place and time, difficulty focusing, short-term memory loss, and increasing lethargy. The nurse suspects which condition in this client? 1. Amnesia (14%) 2. Delirium (72%) 3. Dementia (8%) 4. Psychosis (5%) OmittedCorrect answer 2 72%Answered correctly

Delirium or acute cognitive dysfunction is a syndrome commonly seen in hospitalized clients; it is reversible but difficult to diagnose. Clients may manifest delirium states that can be hypoactive (eg, quiet, disorientation, change in level of consciousness, memory loss), hyperactive (eg, restlessness, agitation, hallucinations, paranoia), or mixed. Manifestations of delirium develop acutely and are difficult to differentiate from those associated with pain, anxiety, and medications. Early diagnosis and treatment are advantageous as delirium is associated with increased mortality (especially in critically ill clients on mechanical ventilation). Delirium is difficult to assess; it is recommended that nurses use a standardized tool (eg, Confusion Assessment Method for the ICU) or checklist (eg, Intensive Care Delirium Screening) for this purpose. (Option 1) Amnesia affects short- and long-term memory loss. It can be intentionally induced by drug use or may occur as a result of trauma or underlying physical/psychological disease processes. Amnesia is not the most likely condition manifested by this client. (Option 3) In contrast to delirium, dementia is gradual in onset and causes an irreversible and progressive cognitive decline. Remote memory is spared initially and there is no impairment of consciousness until the late stages of the disease. (Option 4) Psychosis does not have an acute onset. Clients with this condition are usually oriented but have auditory (not visual) hallucinations. It is not likely in this client. Educational objective:New-onset confusion regarding sense of place and time, difficulty focusing, short-term memory loss, and increasing lethargy can be manifestations of delirium in a critically ill client who was previously alert and oriented.

/The nurse is assessing a 3-month-old during a well-child visit. Which developmental finding should the nurse expect to observe in the client? 1. Infant cries and clings to parent when members of the health team come near (6%) 2. Infant kicks legs, smiles, and coos when a familiar face comes into view (81%) 3. Infant transfers a ball from one hand to the other hand (1%) 4. Infant turns from the back to the abdomen (10%) OmittedCorrect answer 2 81%Answered correctly

Developmental milestones (eg, motor, sensory, verbal, cognitive) are known patterns of growth and development noted in most children by a specific age. These milestones are used as a general assessment guide, although each child has a unique pattern of development. By age 3 months, the infant recognizes familiar items and faces (Option 2). Any 3-month-old who does not respond to familiar faces may have visual impairment or an underlying neurological disorder (eg, autism). (Option 1) Stranger anxiety is part of the infant's normal social and cognitive development and usually begins around age 6 months. (Option 3) Transferring objects from one hand to the other hand is a fine motor skill that usually develops between age 6 and 9 months. Failure to develop this skill may indicate neuromuscular or developmental delays. (Option 4) A 3-month-old is usually not strong enough to roll from the back to the front. Infants should be able to turn from the abdomen to the back at around age 4 months and then from the back to the abdomen by age 6 months. Failure to roll over by age 6 months may indicate slower-than-normal neck, leg, back, and arm muscle development and should be investigated. Educational objective:Developmental milestones (known patterns of growth and development) are used as a general assessment guide. By age 3 months, the infant should recognize familiar items and faces. Any 3-month-old who does not respond to familiar faces (eg, by cooing) may have visual impairment or an underlying neurological disorder (eg, autism).

An adolescent client seen in the ambulatory care center is going on a one-week fasting regimen of water and juice to jump start weight loss. The nurse's response is based on an understanding of which of the following? 1. Fasting for 7 days is not likely to cause health problems (2%) 2. Fasting spares protein in favor of fat metabolism (18%) 3. Fasting will help control hunger pangs in the long term (1%) 4. Initial weight loss during fasting is primarily from fluid loss (77%) OmittedCorrect answer 4 77%Answered correctly

Fasting for more than 1 or 2 days can cause a number of health problems: Increased stress - when fasting, the body goes into "starvation mode;" metabolism slows down and cortisol production increases Muscle damage - in starvation mode, the body breaks down muscle and converts amino acids to glucose Fluid loss - glycogen stores in the liver are also broken down as an energy source; this metabolic process releases water, resulting in fluid loss Increased hunger - appetite hormones are suppressed during a fast; however, when regular eating habits are resumed, appetite will be increased Depletion of essential nutrients Fatigue, headache, dehydration, dizziness, and muscle weakness (Option 1) Fasting for more than 1 or 2 days can lead to health problems. (Option 2) In starvation mode, the body will use protein, fat, and stored carbohydrates as energy sources. Protein is not spared. (Option 3) Appetite may be suppressed during a fast after the first few hours/days; however, when regular intake is resumed, hunger will return. Educational objective:Fasting can cause multiple health problems, including increased stress, slowing of the body's metabolism, muscle damage, fluid loss, increased hunger, depletion of nutrients, and physical symptoms such as headache, dizziness, fatigue, and muscle weakness. Additional Information Basic Care and Comfort NCSBN Client Need

A laboring client at 35 weeks gestation comes to the labor and delivery unit with preterm rupture of membranes "about 18 hours ago." The client's group B Streptococcus status is unknown. What intervention is a priority for this client? 1. Administration of prophylactic antibiotics (46%) 2. Assessment of uterine contraction frequency (20%) 3. Collection of a clean-catch urine specimen (10%) 4. Vaginal examination to assess cervical dilation (21%) OmittedCorrect answer 1 46%Answered correctly

Group B Streptococcus (GBS) may be present as part of normal vaginal flora in up to 30% of pregnant clients. Although colonization with GBS rarely poses harm to the client, it can be transmitted to the newborn during labor and birth, resulting in serious complications (eg, neonatal GBS sepsis, pneumonia). Pregnant clients are tested for GBS colonization at 35-37 weeks gestation and receive prophylactic antibiotics during labor if results are positive. If GBS status is unknown, antibiotics are typically indicated when membranes have been ruptured for ≥18 hours, maternal temperature is ≥100.4 F (38 C), or gestation is <37 weeks (Option 1). (Option 2) Part of the client's assessment includes evaluation of the uterine contraction pattern. However, the client and newborn are at risk for infection due to prolonged rupture of membranes and unknown GBS status, so antibiotic administration is the priority. (Option 3) A urine specimen is often collected to evaluate for proteinuria in clients with elevated blood pressure or to assess for urinary tract infection in symptomatic clients. Urine specimen collection is not the priority for this client. (Option 4) Vaginal examinations should be limited in the presence of ruptured membranes. Multiple vaginal examinations in such a client correlate with an increased risk for infection (eg, chorioamnionitis). Educational objective:Group B Streptococcus (GBS) infection can be transmitted to the newborn during labor and birth and cause serious complications. Indications for prophylactic antibiotics during labor include maternal GBS-positive status or unknown GBS status with fever ≥100.4 F (38 C), preterm gestation, and/or prolonged rupture of membranes. Additional Information Health Promotion and Maintenance NCSBN Client Need

A client with type 2 diabetes, coronary artery disease, and peripheral arterial disease developed hospital-acquired pneumonia (HAP) and has been receiving intravenous (IV) antibiotics for 4 days. Which parameter monitored by the nurse best indicates the effectiveness of treatment? 1. Color of sputum (7%) 2. Lung sounds (37%) 3. Saturation level (6%) 4. White blood cell count (WBC) (48%) OmittedCorrect answer 4 48%Answered correctly

HAP is a bacterial infection acquired in a health care facility that was not present on admission. Almost all clients with bacterial pneumonia develop leukocytosis (WBC >11,000/mm3). Antibiotic therapy is the first-line treatment, but antibiotic resistance frequently occurs in HAP. If antibiotic therapy is effective, clinical improvement usually occurs within 3-4 days of initiation of IV antibiotics. The nurse monitors WBC as the best indicator of treatment effectiveness as antibiotics cause bacterial lysis or hinder bacterial DNA reproduction. The reduced number of bacteria and the resulting decrease in inflammation cause a decrease in the number of white blood cells needed to fight the infection. Other indicators of treatment effectiveness include improvement of infiltrates on chest x-ray, oxygenation, and signs and symptoms (cough, fever, sputum production). (Option 1) The color of sputum (eg, clear, yellow, green, grey, rusty, blood-tinged) can vary with different types of pneumonia; it is not the best indicator of treatment effectiveness. (Option 2) Adventitious/abnormal lung sounds (crackles, low-pitched wheeze, bronchial breath sounds) can be present as the pneumonia resolves or can be a sign of further complication (pleural effusion). However, these are not the best indicators of treatment effectiveness. (Option 3) Saturation is an indicator of oxygenation but can be affected by many other factors, such as coexisting disease, peripheral circulation, and drugs. It is not the best indicator of treatment effectiveness. Educational objective:Indicators of treatment effectiveness for HAP include decreased WBC on complete blood count with differential and improvement of infiltrates on chest-x-ray, oxygenation, and signs and symptoms (cough, fever, sputum production).

The nurse assesses a pediatric client who was diagnosed with diarrhea caused by Escherichia coli. The nurse is most concerned with which finding? 1. Blood-streaked stools (22%) 2. Client drank fruit juice (1%) 3. Dry mucous membranes (28%) 4. Petechiae noted on the trunk (47%) OmittedCorrect answer 4 47%Answered correctly

Hemolytic uremic syndrome (HUS) is a life-threatening complication of Escherichia coli diarrhea and results in red cell hemolysis, low platelets, and acute kidney injury. Hemolysis results in anemia, and low platelets manifest as petechiae or purpura. Therefore, the presence of petechiae in this client could indicate underlying HUS and needs further assessment. (Option 1) E coli bacteria infect people through contaminated food or water and attack the digestive system. Blood-streaked stool due to intestinal irritation is a common symptom associated with this illness. Treatment is aimed at preventing dehydration, and clients usually improve in about a week. (Option 2) Fruit juices are discouraged in acute diarrhea as they have high sugar (osmolality) and low electrolyte content. Continuing the client's normal diet (solid foods) is encouraged as it shortens the duration and severity of the diarrhea. (Option 3) Dry mucous membranes are a sign of dehydration, a common complication of any persistent diarrhea. Dehydration should be treated promptly, especially in children; however, as long as fluid is replenished, the condition is not life-threatening. Educational objective:Hemolytic uremic syndrome is a life-threatening complication of Escherichia coli diarrhea. Clinical features include anemia (pallor), low platelets (petechiae and purpura), and acute kidney injury (low urine output).

/A graduate nurse is reinforcing education to a pregnant client with hepatitis B who expresses concern about transmitting the virus to the newborn after birth. Which statement about newborn care made by the graduate nurse should cause the precepting nurse to intervene? 1. "IM injections will be given after the newborn's bath to reduce exposure to bodily fluids during needle sticks." (34%) 2. "The newborn will receive both the hepatitis B vaccination and hepatitis B immune globulin injection after birth." (20%) 3. "You may safely initiate skin-to-skin contact after birth, which promotes bonding and keeps the newborn warm." (8%) 4. "You will need to formula feed your newborn to reduce the risk of transmitting the virus via breast milk." (36%) OmittedCorrect answer 4 36%Answered correctly

Hepatitis B virus infection is a bloodborne disease that poses a significant infection risk to the newborn because of exposure to maternal blood and bodily fluids during birth. The most important interventions to prevent maternal-to-newborn transmission after birth include initiation of the hepatitis B vaccine series and administration of hepatitis B immune globulin (HBIG) within 12 hours of birth. Clients who desire to breastfeed should be encouraged to do so if possible because very few absolute contraindications to breastfeeding exist. Breastfeeding has not been shown to affect newborn infection rates and is not contraindicated as long as the client's nipples are intact (eg, not bleeding) and immunoprophylaxis (ie, HBIG, hepatitis B vaccine) is appropriately administered (Option 4). (Option 1) To protect the newborn from further exposure to maternal blood and bodily fluids, the nurse should wash the newborn's skin prior to any procedures that puncture the skin (eg, vaccination). (Option 2) All newborns of mothers with a positive hepatitis B surface antigen (HBsAg) test should receive the HBIG injection and hepatitis B vaccination to prevent infection and ensure long-term immunity. (Option 3) Skin-to-skin contact promotes maternal-newborn bonding, breastfeeding initiation, and temperature regulation and is not contraindicated for clients with hepatitis B. Educational objective:Hepatitis B virus infection is a bloodborne disease that poses a significant infection risk to the newborn. It is not a contraindication to breastfeeding. However, the hepatitis B immune globulin and vaccine should be administered to the newborn within 12 hours of birth.

A client taking a diuretic for chronic heart failure experiences constipation. What is the nurse's best recommendation? 1. Drink 2 extra glasses of water with each meal (5%) 2. Exercise for longer periods (0%) 3. Include more fiber in the diet (91%) 4. Take warm baths to relax the abdomen (2%) OmittedCorrect answer 3 91%Answered correctly

Increased consumption of fiber is the best recommendation for a client with heart failure who has constipation. Diuretics may lead to dehydration; however, fluid intake is usually limited for clients with heart failure. This client's history of heart failure is a contraindication to increased fluid intake due to potential fluid overload and increased workload on the heart (Option 1). The nurse should consult the health care provider before recommending a specific amount of fluid intake. (Options 2 and 4) Exercise is important, but clients with heart failure may not exercise for long periods due to fatigue. Warm baths are not helpful for constipation. Educational objective:When a client taking a diuretic experiences constipation, the nurse must consider the reason for taking the diuretic before offering a recommendation. Increased fluid intake is usually contraindicated in clients with a history of heart failure.

Four clients are seen by the emergency department nurse. Which client is a priority for treatment and definitive care? 1. 7-day-old fussy infant with a rectal temperature of 100.6 F (38.1 C) and 6 wet diapers today (41%) 2. Client receiving radiation therapy who has 6-in (15.2-cm) arm laceration that is not actively bleeding (26%) 3. Client with purulent drainage and crusting of the eyelid with vision unaffected (10%) 4. New parent who is crying and overwhelmed, and denies suicidal ideation (21%) OmittedCorrect answer 1 41%Answered correctly

Infants <30 days old have immature immune systems and a blunted response to infection. The 7-day-old infant is at high risk for bacteremia. Infectious manifestations are often subtle at this age (eg, fever can be the only symptom), although some infants may have hypothermia, lethargy, poor feeding, or decreased urine output. Rectal temperature >100.4 F (38.0 C) or <96.8 F (36.0 C) is a "red flag" in a neonate. (Option 2) The client receiving radiation therapy is stable, and there is 6- to 8-hour window in which to safely close the wound. This is not a high-risk client. (Option 3) Bacterial conjunctivitis (pink eye) presents with conjunctival erythema; thick, purulent drainage; and "crusted" eyelids. The client will receive antibiotic drops or ointment, warm soaks/cool compresses, and infection control. Pink eye is highly contagious but not emergent. (Option 4) The parent has postpartum blues/depression and is not emergent. This client can be counseled or provided resources later after the infant with fever is seen. Educational objective:Infectious manifestations are often subtle in neonates (eg, fever can be the only symptom), although some may have hypothermia, lethargy, poor feeding, or decreased urine output. Rectal temperature >100.4 F (38.0 C) or <96.8 F (36.0 C) is a "red flag" in a neonate. Additional Information Management of Care NCSBN Client Need

The clinic nurse is caring for several clients during well-child visits. The nurse should recognize which client as being the most at risk for anemia? 1. 1-month-old infant born at term gestation who exclusively breastfeeds (7%) 2. 2-month-old infant born at preterm gestation who exclusively receives iron-fortified formula (27%) 3. 3-month-old infant born at preterm gestation who is exclusively bottle-fed with breastmilk (62%) 4. 6-month-old infant born at term gestation who breastfeeds and eats iron-fortified infant cereal (2%) OmittedCorrect answer 3 62%Answered correctly

Iron deficiency during infancy causes reduced hemoglobin production, resulting in anemia, decreased immune function, and delayed growth and development. During gestation, the fetus stores iron received from the mother; the amount of iron stored is dependent on the length of gestation. After birth, iron stores are progressively depleted and nutritional sources of iron are eventually required. Infants born at preterm gestation have less time in utero to accumulate iron. Preterm infants typically deplete iron stores by age 2-3 months and require additional iron supplementation (eg, oral iron drops, iron-fortified formula). Therefore, a 3-month-old infant born at preterm gestation who is exclusively receiving breastmilk is most at risk for anemia (Option 3). (Options 1 and 4) Infants born at term gestation have sufficient iron stores for the first 4-6 months of life. However, infants receiving exclusively breastmilk require iron supplementation (eg, oral iron drops) around age 4 months until food sources of iron (eg, iron-fortified infant cereal) are adequate around age 6 months. (Option 2) Although this client is at risk for anemia due to preterm gestation, the risk decreases due to intake of iron-fortified formula. The iron content of most infant formulas is adequate for the first 12 months of life. Educational objective:During gestation, the amount of iron a fetus stores is dependent on the length of gestation. Infants born at preterm gestation have lower iron stores at birth and are at an increased risk for iron-deficiency anemia. Iron supplementation (eg, oral iron drops, iron-fortified formula) is usually needed by preterm infants at an earlier age (2-3 months).

/A client is 1-day postoperative abdominoplasty and is discharged to go home with a Jackson-Pratt (JP) closed-wound system drain in place. The nurse teaches the client how to care for the drain and empty the collection bulb. Which statement indicates that the client needs further instruction? 1. "I'll empty the JP bulb when it is totally full so that I don't have to unplug it so many times." (79%) 2. "I'll pull the plug on the JP bulb and pour the drainage into the measurable specimen cup." (4%) 3. "I'll squeeze the JP bulb from side-to-side as I hold it in my hand." (8%) 4. "While the JP bulb is totally compressed, I'll clean the spout with alcohol and replace the plug." (7%) OmittedCorrect answer 1 79%Answered correctly

It is common for clients to be discharged with a JP closed-wound surgical drain in place after abdominal and breast reconstruction surgery. The purpose of the drain is to prevent fluid buildup in a closed space, which can put tension on the suture line and compromise the integrity of the incision, increase the risk for infection, and decrease wound healing. The general procedure for emptying the drainage device includes the following steps in order: Perform hand hygiene as asepsis must be maintained to prevent the transmission of microorganisms even though there is less chance of bacteria entering the wound using a closed-wound drainage device (eg, JP, Hemovac) than an open-drain device (eg, Penrose) Pull the plug on the bulb to open the device and pour the drainage into a small, calibrated container (eg, plastic water cup, urine specimen container) as this facilitates recording accurate drainage output (Option 2) Empty the device every 4-12 hours unless it is 1/2 to 2/3 full before then because as the small capacity bulb (100 mL) fills, the amount of negative pressure in the bulb decreases (Option 1) Compress the empty bulb by squeezing it from side-to-side with 1 or 2 hands until it is totally collapsed. Although the reservoir can be collapsed by pressing the bottom towards the top, compressing the sides of the reservoir (bulb) is recommended as it is more effective in establishing negative pressure (Option 3) Clean the spout on the bulb with alcohol and replace the plug when it is totally collapsed to restore negative pressure (Option 4) Educational objective:A closed-wound drain device (eg, JP, Hemovac) is used to prevent fluid buildup at the surgical wound site and promote healing. Empty the device every 4-12 hours unless it is 1/2 to 2/3 full before then. Drainage tube patency and negative pressure in the reservoir (bulb) must be maintained to provide adequate drainage. Additional Information Reduction of Risk Potential NCSBN Client Need

The nurse caring for a client with pulmonary edema responds to the mechanical ventilator high-pressure alarm. The nurse would assess for which conditions that can trigger the high-pressure alarm? Select all that apply. 1. Biting endotracheal tube 2. Disconnected ventilator tubing 3. Endotracheal tube cuff leak 4. Excessive airway secretions 5. Kinked ventilator tubing OmittedCorrect answer 1,4,5 54%Answered correctly

Mechanical ventilator alarms (eg, high- or low-pressure limit) alert the nurse to potential problems caused by a change in the client's condition, a problem with the artificial airway (eg, endotracheal or tracheostomy tube), and/or a problem with the ventilator. Peak airway pressure is the amount of pressure required to deliver a tidal volume. Any condition that increases the peak airway pressure can trigger the ventilator high-pressure limit alarm. When this alarm sounds, the nurse should assess for conditions that increase airway resistance and/or decrease lung compliance, such as: Excessive secretions: Obstruct the airway, increasing resistance (Option 4) Biting the endotracheal tube and kinked ventilator tubing: Air flow is obstructed, increasing resistance (Options 1 and 5) (Options 2 and 3) Any condition that decreases airway resistance (eg, tubing disconnect, extubation, endotracheal or tracheostomy tube cuff leak) can trigger the low-pressure limit alarm. Educational objective:When the mechanical ventilator high-pressure limit alarm sounds, the nurse should assess for causes of increased airway resistance in the client (eg, bronchospasm), artificial airway (eg, excessive secretions, biting the endotracheal tube), and/or ventilator system (eg, kinked tubing), as well as for causes of decreased lung compliance (eg, pneumothorax). Additional Information Physiological Adaptation NCSBN Client Need

The nurse provides teaching about methotrexate to a client with rheumatoid arthritis. It is most important to address which topic regarding this drug? 1. Need for an eye examination (13%) 2. Need for sunblock (10%) 3. Risk for infection (50%) 4. Risk for kidney injury (25%) OmittedCorrect answer 3 50%Answered correctly 01 secTime Spent 03/05/2020Last Updated

Methotrexate (Rheumatrex) is classified as a folate antimetabolite, antineoplastic, immunosuppressant drug used to treat various malignancies and as a nonbiologic disease-modifying antirheumatic drug (DMARD) used to treat rheumatoid arthritis and psoriasis. Methotrexate can cause bone marrow suppression resulting in anemia, leukopenia, and thrombocytopenia. Leukopenia and its immunosuppressant effects can increase susceptibility to infection. Clients should be educated about obtaining routine killed (inactivated) vaccines (eg, influenza, pneumococcal) and avoiding crowds and persons with known infections. Live vaccines (eg, herpes zoster) are contraindicated in clients receiving immunosuppressants, such as methotrexate. Alcohol should be avoided in clients taking methotrexate as it is hepatotoxic and drinking alcohol increases the client's risk for hepatotoxicity. (Option 1) Regular eye examinations every 6 months are indicated for clients prescribed the nonbiological DMARD antimalarial hydroxychloroquine (Plaquenil) as it can cause retinal damage. Ethambutol, used to treat tuberculosis, also requires frequent eye examinations. (Options 2 and 4) Photosensitivity (common with tetracycline, thiazide diuretics, and sulfonamides) and nephrotoxicity (common with aminoglycosides, vancomycin, and nonsteroidal anti-inflammatory drugs) can occur, but immunosuppression is more likely and potentially fatal. Educational objective:Methotrexate is a nonbiologic disease-modifying antirheumatic drug used to treat rheumatoid arthritis. The major adverse effects associated with its use include bone marrow suppression, hepatotoxicity, and gastrointestinal irritation.

The nurse observes a student nurse administer ear drops to an elderly client to help loosen cerumen. The nurse intervenes when the student performs which action? 1. Instills ear drops at room temperature (1%) 2. Instills ear drops with dropper by occluding the ear canal (69%) 3. Places a cotton ball loosely in outermost auditory canal after the instillation (14%) 4. Pulls pinna up and back and instills drops (14%) OmittedCorrect answer 2 69%Answered correctly

Otic medications are used to treat infection, soften cerumen for later removal, and facilitate removal of an insect trapped in the ear canal. They are contraindicated in a client with a perforated eardrum. The general procedure for instilling ear drops includes the following steps: Perform hand hygiene and don clean gloves. The ear canal is not sterile, but aseptic technique is used Position the client side-lying with the affected ear up (if not contraindicated). This facilitates administration and prevents drops from leaking out of the ear Warm ear drops to room temperature (ie, use hand or warm water) to help avoid vertigo, dizziness, or nausea as the internal ear is sensitive to temperature extremes (Option 1) Pull the pinna up and back to straighten the ear canal in clients >4 years old and adults. Pull the pinna down and back in clients <3 years old (Option 4) Support hand on the client's head and instill the prescribed number of drops by holding the dropper 1 cm (1/2 in) above the ear canal. This avoids damaging the ear canal with the dropper (Option 2) Apply gentle pressure to the tragus (fleshy part of external ear canal) if it does not cause pain, which facilitates the flow of medication into the ear canal Instruct the client to remain side-lying for at least 2-3 minutes to facilitate medication distribution and prevent leakage Place a cotton ball loosely in the client's outermost ear canal for 15 minutes, only if needed, to absorb excess medication. Perform this with caution and avoid in infants or very young clients as it is a choking hazard (Option 3) Educational objective:To administer otic medications in an adult client, follow these steps: (1) Perform hand hygiene, (2) position the client side-lying with the affected ear up, (3) pull pinna up and back, (4) administer prescribed number of ear drops, (5) instruct the client to remain side-lying for 2-3 minutes, and (6) place cotton ball loosely in the outer ear canal for 15 minutes (if needed). Additional Information Pharmacological and Parenteral Therapies NCSBN Client Need

The nurse reviews the chart of a client who gave birth 4 hours ago. Which contributing factor indicates that the client has an increased risk of postpartum hemorrhage? 1. Infant birth weight of 9 lb 2 oz (4139 g) (58%) 2. Labor and birth without pain medication (1%) 3. Labor that lasted 8 hours (19%) 4. Third stage of labor lasting 20 minutes (19%) OmittedCorrect answer 1 58%Answered correctly

Postpartum hemorrhage (PPH) is usually defined as maternal blood loss of >500 mL after a vaginal birth or >1000 mL after a cesarean birth. Uterine atony, characterized by a soft, "boggy," and poorly contracted uterus, is the most common cause of early PPH (occurring ≤24 hours after birth). Delayed PPH (>24 hours after birth) usually results from retained placental fragments associated with a long third stage of labor (ie, time from birth of baby to expulsion of placenta, lasting >30 minutes). Risk factors for PPH include: History of PPH in prior pregnancy Uterine distension due to:Multiple gestationPolyhydramnios (ie, excessive amniotic fluid)Macrosomic infant (≥8 lb 13 oz [4000 g]) (Option 1) Uterine fatigue (labor lasting >24 hours) High parity Use of certain medications:Magnesium sulfateProlonged use of oxytocin during laborInhaled anesthesia (ie, general anesthesia) (Option 2) Natural, unmedicated labor and birth reduces the chance of PPH. (Option 3) Labor lasting <24 hours does not increase the risk for PPH. (Option 4) A third stage of labor lasting <30 minutes does not increase the risk for PPH. Educational objective:Postpartum hemorrhage is defined as maternal blood loss of >500 mL after a vaginal birth or >1000 mL after a cesarean birth. Uterine atony (ie, "boggy" uterus) is the most common cause of early postpartum hemorrhage (occurring ≤24 hours after birth). Risk factors include uterine distension, uterine fatigue, high parity, and certain medications.

A client who gave birth vaginally with epidural anesthesia still has limited movement and strength of the right leg, and reports no urge to urinate at 2 hours postpartum. The nurse palpates the client's fundus 2 cm above the umbilicus and to the right. What should the nurse do next? 1. Assist the client to the bathroom in a wheelchair (42%) 2. Encourage the client to drink plenty of fluids (5%) 3. Perform in-and-out catheterization (39%) 4. Reassess for bladder distension hourly (12%) OmittedCorrect answer 3 39%Answered correctly

Postpartum urinary retention is commonly related to decreased bladder sensation (eg, due to regional anesthesia, prolonged labor, or perineal trauma) and postpartum diuresis. Urinary retention can cause bladder distension, which may be noted by a displaced and/or boggy uterus, or by a palpable bladder. If bladder distension cannot be resolved with spontaneous voiding, in-and-out (I&O) catheterization may be indicated, especially if the client: Is unable to ambulate to the restroom or void into a bedpan (Option 3) Has not voided within 6-8 hours after delivery or removal of the indwelling urinary catheter after cesarean delivery Has difficulty emptying bladder completely (ie, voiding <100 mL frequently) (Option 1) Postpartum clients are at risk for falls due to regional or general anesthesia, orthostatic hypotension, and changes in center of gravity. Risk of injury is increased in clients who ambulate before full sensory/motor function has returned. (Option 2) Fluid intake may be encouraged postpartum but does not resolve the client's urinary retention and bladder distension. (Option 4) A displaced fundus (ie, elevated above the umbilicus and/or to one side) is evidence of bladder distension. Reassessing in an hour may increase the risk of postpartum hemorrhage, as a full bladder can cause uterine atony. Educational objective:Signs of bladder distension in a postpartum client include a displaced and/or boggy uterus, or a palpable bladder. Performing an in-and-out catheterization can help prevent postpartum hemorrhage related to uterine atony and injuries related to falls if sensory/motor function of the lower extremities is decreased.

The nurse is reinforcing instructions to a client being discharged from the clinic with a diagnosis of acute prostatitis. Which statement by the client indicates an understanding of the instructions? 1. "Having sex will make the infection worse." (13%) 2. "I enjoy iced tea, so I will drink more to stay hydrated." (8%) 3. "I should take ciprofloxacin until I feel better." (9%) 4. "I should take docusate to prevent straining." (68%) OmittedCorrect answer 4 68%Answered correctly

Prostatitis is inflammation of the prostate gland, usually caused by a bacterial infection. Symptoms include rectogenital pain, burning, urinary hesitancy, and/or urinary urgency. Management of prostatitis includes antimicrobial and anti-inflammatory medications (eg, ibuprofen). Alpha-adrenergic blockers (eg, tamsulosin, alfuzosin) help relax the bladder and prostate. Suprapubic catheterization may be necessary for urinary retention in severe cases of acute prostatitis. Urethral catheterization is contraindicated due to the risk of exacerbating pain and urethral inflammation. Clients should be instructed to: Hydrate with clear liquids (eg, water, fruit juices). Avoid coffee, tea, and other caffeinated beverages due to diuretic and stimulant properties, which may worsen symptoms (Option 2). Complete the full course of antibiotics regardless of symptom improvement to ensure infection resolution (Option 3). Engage in sexual intercourse or masturbation to reduce discomfort related to retained prostatic fluid. Clients should use a barrier prophylactic method (eg, condoms) when engaging in sexual activity with a partner to prevent transmission of the causative organism (Option 1). Take stool softeners as prescribed to reduce straining during defecation; tension of the pubic muscles presses against the prostate, causing pain (Option 4). Take sitz baths, in which the hips and buttocks are immersed in warm water, to help relieve symptoms. Educational objective:Clients with prostatitis should be instructed to take anti-infective and anti-inflammatory medications, stool softeners, and sitz baths and to decrease retained prostatic fluid via ejaculation. Hydration and avoidance of caffeine should also be promoted.

/The nurse cares for a client with a pulmonary embolism. Which of the following clinical manifestations would the nurse anticipate? Select all that apply. 1. Bradycardia 2. Chest pain 3. Dyspnea 4. Hypoxemia 5. Tachypnea 6. Tracheal deviation OmittedCorrect answer 2,3,4,5 60%Answered correctly

Pulmonary embolism (PE) is a potentially life-threatening medical emergency occurring when a blood clot, fat or air embolus, or tissue (eg, tumor) travels via the venous system into the pulmonary circulation and obstructs blood flow into the lung. This prevents deoxygenated blood from reaching the alveoli, which leads to hypoxemia due to impaired gas exchange and cardiac strain due to congested blood flow in the pulmonary arteries. Clinical manifestations of PE range from mild (eg, anxiety, cough) to severe (eg, heart failure, sudden death). However, many clients initially have mild, nonspecific symptoms that are often misdiagnosed and inadequately managed, greatly increasing the likelihood of progression to shock and/or cardiac arrest. Clinical manifestations of PE include: Pleuritic chest pain (ie, sharp lung pain while inhaling) (Option 2) Dyspnea and hypoxemia (Options 3 and 4) Tachypnea and cough (eg, dry or productive cough with bloody sputum) (Option 5) Tachycardia Unilateral leg swelling, erythema, or tenderness related to deep vein thrombosis (Option 1) Tachycardia, rather than bradycardia, is expected with PE because the heart attempts to compensate for hypoxemia, right ventricular overfilling, and decreased left ventricular cardiac output. (Option 6) Tracheal deviation is a sign of tension pneumothorax (not PE), which occurs when pressure on the side of the collapsed lung pushes organs toward the unaffected lung. Educational objective:Pulmonary embolism is a potentially life-threatening medical emergency occurring when a pulmonary artery is obstructed. Common clinical manifestations include pleuritic chest pain, dyspnea, hypoxemia, tachypnea, cough, tachycardia, and unilateral leg swelling.

A homeless man known to have chronic alcoholism and who has not eaten for 8 days is undergoing nutritional rehabilitation via oral and enteral feedings. Which of the following findings would indicate that the client is developing refeeding syndrome? 1. Phosphorus 2.0 mg/dL (0.65 mmol/L), potassium 2.9 mEq/L (2.9 mmol/L), magnesium 1.0 mEq/L (0.5 mmol/L) (43%) 2. Phosphorus 4.0 mg/dL (1.29 mmol/L), potassium 3.5 mEq/L (3.5 mmol/L), magnesium 2.0 mEq/L (1.0 mmol/L) (11%) 3. Random blood glucose 60 mg/dL (3.3 mmol/L), sodium 120 mEq/dL (120 mmol/L), calcium 7.0 mg/dL (1.75 mmol/L) (28%) 4. Random blood glucose 100 mg/dL (5.6 mmol/L), sodium 140 mEq/dL (140 mmol/L), calcium 10.0 mg/dL (2.50 mmol/L) (17%) OmittedCorrect answer 1 43%Answered correctly

Refeeding syndrome is a potentially lethal complication of nutritional replenishment in significantly malnourished clients and can occur with oral, enteral, or parenteral feedings. After a period of starvation, carbohydrate-rich nutrition (glucose) stimulates insulin production along with a shift of electrolytes from the blood into tissue cells for anabolism. The key signs of refeeding syndrome are rapid declines in phosphorous, potassium, and/or magnesium (mnemonic PPM). Other findings may include fluid overload, sodium retention, hyperglycemia, and thiamine deficiency. Actions to prevent refeeding syndrome include the following: Obtaining baseline electrolytes Initiating nutrition support cautiously with hypocaloric feedings Closely monitoring electrolytes Increasing caloric intake gradually (Option 2) These values are within normal ranges for phosphorus (2.4-4.4 mg/dL [0.78-1.42 mmol/L]), potassium (3.5-5.0 mEq/L [3.5-5.0 mmol/L]), and magnesium (1.5-2.5 mEq/L [0.75-1.25 mmol/L]). In refeeding syndrome, the values for one or more of these electrolytes are decreased. (Option 3) These laboratory values are below normal ranges but are not associated with refeeding syndrome. (Option 4) These are normal laboratory values and are not associated with refeeding syndrome. Educational objective:Refeeding syndrome is a serious complication of nutritional replenishment. It is marked by declines in serum phosphorus, potassium, and/or magnesium (mnemonic PPM). Clients can also develop fluid overload. Low-calorie feedings and a gradual increase in calories can prevent refeeding syndrome. Electrolytes should be monitored frequently. Additional Information Reduction of Risk Potential NCSBN Client Need

Which are appropriate examples of cost-effective care? Select all that apply. 1. Considering the inside of the sterile glove wrapper as a small sterile field 2. Donning clean, rather than sterile, gloves to remove a client's dressing 3. Returning opened, unused supplies from a client's room to the central supply room 4. Reusing a tourniquet for multiple clients unless it is visibly soiled 5. Using remaining sterile saline in a bottle opened 48 hours ago before discarding OmittedCorrect answer 1,2 32%Answered correctly

Removing a dressing that has been on the client's skin is not a sterile procedure (unlike applying a new dressing, when sterile technique is commonly used). The gloves need to be removed and changed prior to application of a new dressing. There is no need to use the more expensive sterile gloves. The sterile glove wrapper is inside a paper package and is sterile. It can be used as a small sterile field if properly opened, with the other aspects of asepsis/sterile field observed (eg, do not get it wet, do not reach over it). (Option 3) Once supplies have been in a client's room, they are "contaminated" and cannot be returned to a central source or used on other clients. They can be sent home with the client for the client's own use. (Option 4) Tourniquets should be for single client use. They should not be shared as there is a risk of cross-infection, even if contamination is not visible. (Option 5) Pour bottles and IV bags with sterile solutions have no preservative and must be changed every 24 hours after being accessed. After 24 hours following opening a bottle of sterile saline, the solution is considered contaminated and must be discarded, even if there is still unused solution in the container. Educational objective:Use clean, rather than sterile, gloves when removing contaminated dressings. The inside of a sterile glove wrapper is sterile. Do not return items in clients' rooms to central supplies, discard sterile solution after 24 hours, and do not reuse tourniquets between clients. Additional Information Management of Care NCSBN Client Need

/A nurse is participating in an obstetrical emergency simulation in which the health care provider announces shoulder dystocia. Which of the following interventions should the assisting nurse implement? Select all that apply. 1. Assist maternal pushing efforts by applying fundal pressure during each contraction 2. Document the time the fetal head was born 3. Flex the client's legs back against the abdomen and apply downward pressure above the symphysis pubis 4. Prepare for a forceps-assisted birth 5. Request additional assistance from other nurses immediately OmittedCorrect answer 2,3,5 24%Answered correctly

Shoulder dystocia is an unpredictable obstetrical emergency that occurs during vaginal birth when the fetal head delivers but the anterior (top) shoulder becomes wedged behind or under the mother's symphysis pubis. Shoulder dystocia lasting ≥5 minutes is correlated with almost certain fetal asphyxia resulting from prolonged compression of the umbilical cord. Minimizing the time it takes to deliver the fetal body is essential for reducing adverse outcomes (eg, hypoxia, nerve injury, death). When shoulder dystocia occurs, the primary nursing interventions include: Documenting the exact time of events (eg, birth of fetal head, shoulder dystocia maneuvers) (Option 2) Verbalizing passing time to guide decision-making by the health care provider (eg, "two minutes have passed") Performing maneuvers to relieve shoulder impaction (eg, McRoberts maneuver, suprapubic pressure) (Option 3) Requesting additional help from staff (eg, nurses, neonatologist) immediately (Option 5) (Options 1 and 4) Fundal pressure and the use of forceps or a vacuum to facilitate birth are contraindicated because they may further wedge the fetal shoulder into the maternal symphysis pubis and increase the risk for neurological complications (eg, brachial plexus injury) in the newborn. Educational objective:Shoulder dystocia occurs when the anterior shoulder becomes wedged behind or under the maternal symphysis pubis. The nurse should document the timing of events (eg, birth of fetal head), verbalize passing time, perform McRoberts maneuver, apply suprapubic pressure, and request additional assistance. Additional Information Physiological Adaptation NCSBN Client Need

A nurse is teaching a client with a surgically repaired undescended testis about testicular self-examination (TSE). Which instructions should be included in the teaching? Select all that apply. 1. Perform the examination during a warm bath or shower 2. Perform the examination monthly on the same day 3. Report if one testis is slightly larger than the other 4. Report if there is a hard mass over the testis 5. Use both hands to feel each testis separately OmittedCorrect answer 1,2,4,5 25%Answered correctly

Testicular cancer is the most common form of cancer in men age 15-35. When diagnosed early, it is highly curable. Clients at high risk for developing a tumor (eg, history of undescended testis) are encouraged to perform a monthly TSE. Client instructions for a TSE include: Perform TSE monthly on the same day (easy to remember) Perform TSE while taking a warm shower or bath as warm temperatures will relax the scrotal tissue and make the testis hang lower in the scrotum Use both hands to feel each testis separately Palpate each testicle gently, using the thumb and first 2 fingers Check that the testicle is normally egg-shaped and movable with a smooth surface The clinical findings that should be reported to the health care provider include: Painless, hardened lump on testes Scrotal swelling or heaviness Dull ache in pelvis or scrotum (Option 3) It is normal for one testicle to be slightly larger or hang lower than the other. Some people may also confuse epididymis (small coiled tube) as a small lump at the beginning. These do not need to be reported. Educational objective:Clients with a history of undescended testis or testicular tumor are encouraged to perform a monthly TSE. It is best done during a warm shower. The first indication of testicular cancer may be a painless, hardened lump on the testes or a feeling of scrotal heaviness. Additional Information Health Promotion and Maintenance NCSBN Client Need

//A nurse is admitting a client at 42 weeks gestation to the labor and delivery unit for induction of labor. What is a predictor of a successful induction? 1. Bishop score of 10 (59%) 2. Firm and posterior cervix (10%) 3. History of precipitous labor (6%) 4. Reactive nonstress test (23%) OmittedCorrect answer 1 59%Answered correctly

The Bishop score is a system for the assessment and rating of cervical favorability and readiness for induction of labor. The cervix is scored (0-3) on consistency, position, dilation, effacement, and station of the fetal presenting part. A higher Bishop score indicates an increased likelihood of successful induction that results in vaginal birth. For nulliparous women, a score ≥6-8 usually indicates that induction will be successful (Option 1). (Option 2) A cervix that is firm and posterior is associated with a low Bishop score, which reflects a low likelihood of successful labor induction. (Option 3) A history of precipitous labor (<3 hours from onset of contractions to birth) may indicate that the client will again experience precipitous labor once labor is established. However, such a history is not an independent predictor of successful induction. (Option 4) A reactive nonstress test indicates that the fetus is well oxygenated and establishes fetal well-being. It does not provide information about the likely success or failure of labor induction. Educational objective:The Bishop score is a system for the assessment and rating of cervical favorability and readiness for induction of labor. A score ≥6-8 in nulliparous women is associated with successful induction and subsequent vaginal birth. Additional Information Health Promotion and Maintenance NCSBN Client Need

A nursing unit implements a quality improvement process of written reminders to ameliorate incentive spirometer (IS) use in postoperative clients. What is the best indicator that the client goal for this process has been met? 1. Chart audits indicate that client incidence of nosocomial pneumonia decreased by 20% (79%) 2. Documentation shows that 100% of nurses attended an inservice seminar on the topic (2%) 3. Nurses report an increased number of written reminders given to appropriate clients (7%) 4. Surgeons who admit to the unit report increased satisfaction with current client IS use (9%) OmittedCorrect answer 1 79%Answered correctly

The best indicators of a successful intervention (desired effect achieved) are objective criteria. This is an objective measurable result that can be correlated with the intervention. (Option 2) Attending an inservice seminar for staff education is an important and necessary step for intervention implementation. However, the intervention will be successful only if the information is applied and the desired outcome achieved. (Option 3) Reporting the number of written reminders given to respective clients is necessary. However, this reporting of intervention achievement is subjective as recall can be inaccurate. Even if it were an accurate recounting, it does not prove that the intervention succeeded. The appropriate focus should be on client outcomes, not nursing staff behaviors. (Option 4) Although approval from surgeons provides helpful support for the intervention, an objective evaluation beyond personal opinions is required. Educational objective:The effectiveness of an intervention should be determined by objective measurable outcomes that can be correlated with the intervention. It should not be based only on personal opinion or staff activities.

A client with end-stage liver disease is admitted for a transplant workup. The client's spouse states that the client has not stopped drinking alcohol and may be unable to quit for 6 months before the transplant. Which is the most appropriate action for the nurse to implement? 1. Ask the transplant team to place a palliative care referral so the client can learn about the option of hospice instead of transplant (1%) 2. Assess the client's motivation to make the necessary self-care changes before and after the transplant (87%) 3. Schedule a meeting to enlist the help of family members in encouraging the client to stay sober until the transplant (2%) 4. Tell the nurse manager that the client may not be an appropriate transplant candidate (9%) OmittedCorrect answer 2 87%Answered correctly

The client may not be an appropriate transplant candidate due to his alcohol use. However, additional facts are needed to determine the true situation as the only information obtained came from the client's spouse. The nurse should assess the client's drinking habits and motivation to stop drinking before and after the transplant by speaking with the client directly. In addition, a transplant requires many other self-care regimens. The nurse should be alert for indicators of the client's ability to take prescribed medications, follow dietary restrictions, and attend medical appointments. The information obtained from this assessment should be communicated to the interdisciplinary team members responsible for determining transplant eligibility. (Option 1) Transplant and hospice care are options for the client with end-stage liver disease; palliative specialists can assist with identifying the goals of care and facilitating decision-making. However, this does not directly address the concern of the client's spouse. At this time, further assessment is necessary before appropriate interventions can be planned. (Option 3) At this point, it is unclear whether the client is motivated to quit drinking (the nurse has only heard from the spouse). Therefore, it is premature to plan for this type of intervention. The client will need to stay sober not just prior to the surgery, but for a lifetime if the transplant occurs. Any discussion between the client's family and the medical team should be based on this understanding. (Option 4) If the nurse is concerned about transplant eligibility due to the spouse's statement, then this should be communicated to the client's interdisciplinary team after further assessment. At this point, it is premature to involve the nurse manager. Educational objective:The client with end-stage organ failure must be motivated and able to engage in complex self-care regimens before and after solid organ transplant. Concerns about the client's motivation to engage in necessary self-care requirements (eg, alcohol abstinence) require further assessment. Additional Information Management of Care NCSBN Client Need

The nurse is caring for a client who weighs 450 lb (204.1 kg) 2 days after bariatric surgery. The client is pleasant, cooperative, and able to fully bear weight. What would be the most appropriate method for transferring this client safely? 1. 1-person safety standby with walker (41%) 2. 2-person full-body sling lift (9%) 3. 2-person standing-assist lift (34%) 4. 4-person full-body sling lift (14%) OmittedCorrect answer 1 41%Answered correctly

When determining the most appropriate method to transfer a client safely, the nurse should assess: Whether the client can bear weight Whether the client is cooperative This client is able to bear full weight despite having a heavy body and can cooperate during the transfer. Therefore, such clients should be encouraged to do as much as they can for themselves, anticipating discharge in the near future. It is appropriate to transfer this client with 1 person standing by for safety. If the client was unable to bear full weight, more assistance would be needed. The number of caregivers providing assistance during the transfer of a heavier client should be increased to promote safety for the client and staff. When working with bariatric clients, equipment that has the capacity to bear the client's full weight and accommodate their size should be used while maintaining the client's dignity throughout the process. (Options 2 and 3) These would not be necessary as this client can fully bear weight and cooperate with caregiver instructions during the transfer. (Option 4) A 4-person sling lift transfer is appropriate for the bariatric client who cannot bear weight or cooperate with the transfer. Educational objective:A client who is able to fully bear weight and cooperate can transfer independently with standby assistance for safety. If there is any concern for caregiver or client safety during the transfer of a bariatric client, the type of equipment should be reconsidered and the number of caregivers should be increased. Additional Information Safety and Infection Control NCSBN Client Need

exhibit//Laboratory results PH 7.25 PO2 79 mm Hg (10.5 kPa) PaCO 235 mm Hg (4.66 kPa) HCO3-12 mEq/L (12 mmol/L) A client's arterial blood gases (ABGs) are shown in the exhibit. The nurse would expect which finding to demonstrate that the client is compensating for the ABGs? Click on the exhibit button for additional information. 1. Decrease in bicarbonate reabsorption (26%) 2. Decrease in respiratory rate (14%) 3. Increase in bicarbonate reabsorption (21%) 4. Increase in respiratory rate (37%) OmittedCorrect answer 4 37%

The client's ABGs have low pH consistent with acidosis. If it is a primary respiratory acidosis, pCO2 would be higher. If it is metabolic acidosis, bicarbonate would be lower. Because this client has low pH coupled with low bicarbonate, the most likely diagnosis is primary metabolic acidosis. Respiratory alkalosis is the body's natural compensation for metabolic acidosis. Respiratory alkalosis is achieved by blowing more CO2 off from the system through rapid breathing. (Option 1) Decreased bicarbonate reabsorption would produce metabolic acidosis; this would occur as a compensation for primary respiratory alkalosis (decreased pCO2 and high pH). (Option 2) When the respiratory rate is decreased, pCO2 would increase, creating a respiratory acidosis; this would occur in response to a primary metabolic alkalosis. (Option 3) Increased bicarbonate reabsorption would produce metabolic alkalosis; this would occur as a compensation for primary respiratory acidosis (increased pCO2 and low pH). Educational objective:Respiratory alkalosis is the body's natural compensation for metabolic acidosis. It is achieved by blowing more CO2 off from the system through rapid breathing. Additional Information Reduction of Risk Potential NCSBN Client Need

The community health nurse is preparing to teach a group of African American women about prevention of diseases common to their ethnic group. Based on the incidence of disease within this group, which disorders should the nurse plan to discuss? Select all that apply. 1. Cervical cancer 2. Hypertension 3. Ischemic stroke 4. Osteoporosis 5. Skin melanoma OmittedCorrect answer 1,2,3 33%Answered correctly

The incidence of cervical cancer is higher among Hispanics, American Indians, and African Americans. The mortality rate for cervical cancer among African American women is twice as high as that for white American women (Option 1). African Americans have the highest incidence of hypertension in the world, and this condition is more prevalent among the women than men in this ethnic group. The mortality rate for hypertension among African American women is higher than that for white American women (Option 2). African Americans have a higher incidence of ischemic stroke than whites or Hispanics. Risk factors for stroke are related to an increased rate of hypertension, diabetes mellitus, and sickle cell anemia (Option 3). (Option 4) White and Asian women have a higher incidence of osteoporosis than African Americans, but the disease affects all ethnic groups. (Option 5) Melanoma of the skin is more common in people who are of white ancestry, light-skinned, and over age 60 with frequent sun exposure. The incidence of melanoma is 10 times higher in white Americans than African Americans. Educational objective:African Americans have the highest incidence of hypertension in the world as well as increased incidence of stroke and cervical cancer. Whites have a high incidence of osteoporosis and skin cancer (melanoma). Additional Information Health Promotion and Maintenance NCSBN Client Need

//The clinic nurse cares for a 4-year-old who has been diagnosed with a pinworm infection. Which client symptom supports this diagnosis? 1. Anal itching that is worse at night (78%) 2. Intestinal bleeding with anemia (1%) 3. Poor appetite with weight loss (7%) 4. Red, scaly, blistered rings on skin (12%) OmittedCorrect answer 1 78%Answered correctly

The most common worm infection in the United States is pinworm, which is easily spread by inhaling or swallowing microscopic pinworm eggs, which can be found on contaminated food, drink, toys, and linens. Once eggs are ingested, they hatch in the intestines. During the night, the female pinworm lays thousands of microscopic eggs in the skinfolds around the anus, resulting in anal itching and troubled sleep. When the infected person scratches, eggs are transferred from the fingers and fingernails to other surfaces. Pinworm infection is treated with anti-parasitic medications. (Option 2) Hookworms (eg, Ancylostoma) are parasitic bloodsucking roundworms that are contracted from larvae in contaminated soil. They can infect the intestines, causing intestinal bleeding and anemia. (Option 3) Poor appetite, inadequate absorption of nutrients from food, and weight loss are symptoms associated with tapeworm infection (eg, Taenia solium). Tapeworm larvae are ingested when a person eats food that is contaminated with feces or undercooked meat from an infected animal. (Option 4) Ringworm is a skin infection caused by a fungus. It leads to red, scaly, blistered rings on the skin or scalp that grow outward as infection spreads. The fungus is easily spread by sharing hair care instruments and hats or via towels, linens, clothing, and sports equipment. Educational objective:The most common worm infection is pinworm, which is spread by inhaling or swallowing microscopic pinworm eggs, which travel to and hatch in the intestines. During the night, the female pinworm lays eggs in the skinfolds around the anus, resulting in anal itching and disturbed sleep.

///The nurse assesses a newly admitted adult client on a neurological inpatient unit. Which assessment findings require immediate follow-up by the nurse? Select all that apply. 1. Cannot flex the chin toward the chest 2. Eyes move in opposite direction of head when head is turned to side 3. New onset of right arm drift 4. Pupils 8 mm in diameter bilaterally 5. Toes point downward when sole of foot is stimulated OmittedCorrect answer 1,3,4 20%Answered correctly

The nurse performs a neurological assessment to evaluate for changes in motor and sensory functions by assessing pupils, cranial nerves, and neuromuscular functioning. The neck should be supple and able to be flexed toward the chest. Nuchal rigidity requires follow-up due to possible meningeal irritation related to infection (eg, meningitis) (Option 1). A new-onset unilateral drift of a limb could indicate a stroke. The nurse assesses for other strokelike symptoms (eg, changing level of consciousness, asymmetrical smile, garbled speech), activates the facility stroke protocol, and notifies the health care provider accordingly (Option 3). Normal pupils are 3-5 mm in diameter. Pupil dilation can be the result of medication use or neurological causes (eg, increased intracranial pressure, brain herniation) (Option 4). (Option 2) Oculocephalic reflex (doll's eyes) is an expected finding indicating an intact brainstem. It is tested by rotating the head and watching for the eyes to move simultaneously in the opposite direction. The test is not performed if spinal trauma is suspected. (Option 5) The normal finding in adults is an absent Babinski reflex (ie, toes point downward with stimulus to the sole). The presence of Babinski reflex (ie, toes fan outward and upward with stimuli) is expected in infants up to age 1, but in an adult may indicate a brain or spinal cord lesion. Educational objective:Abnormal neurological assessments include nuchal rigidity; new-onset unilateral drift of a limb; pupils <3 mm or >5 mm in diameter; absent oculocephalic reflex; and presence of Babinski reflex in an adult client.

A client who is 24 hours postoperative bowel resection is receiving IV opioids PRN for severe pain. The nurse reviews the health care provider's prescription to discontinue the continuous IV normal saline. What is the nurse's most appropriate action? 1. Convert to a saline lock (55%) 2. Remove the IV catheter (1%) 3. Request a prescription for a saline lock (28%) 4. Slow the IV fluids to a keep-vein-open rate (14%) OmittedCorrect answer 1 55%Answered correctly

The nurse should discontinue the IV infusion of normal saline and apply a saline lock to maintain IV access while preventing clotting. The prescription of the health care provider (HCP) to lock the IV catheter is implied, as the client is currently receiving PRN IV opioids (Option 1). A saline lock is sufficient to maintain the line patency and allows greater mobility than a continuous infusion. (Option 2) The client is only 24 hours postoperative abdominal surgery, so IV access is necessary to administer medications (eg, antibiotics, analgesics, antiemetics). (Options 3 and 4) The HCP's prescription specifies discontinuing IV fluids but not removing the IV catheter or slowing the infusion to a keep-vein-open (KVO) rate. Also, the nurse would need to clarify a KVO prescription with the HCP for a precise rate. Educational objective:IV access is necessary for administering intermittent IV opioids to control postoperative pain. A saline lock keeps the line patent and allows greater mobility than a continuous infusion. Additional Information Pharmacological and Parenteral Therapies NCSBN Client Need

/The nurse moves a finger in a horizontal and vertical motion in front of the client's face while directing the client to follow the finger with the eyes. Which cranial nerves is the nurse assessing? Select all that apply. 1. II 2. III 3. IV 4. V 5. VI OmittedCorrect answer 2,3,5 18%Answered correctly

The oculomotor (cranial nerve III), trochlear (cranial nerve IV), and abducens (cranial nerve VI) are motor nerves of the eye that are tested by having the client track an object, such as a finger, through the fields of vision. The oculomotor nerve is also tested by checking for pupillary constriction and accommodation (constriction with near vision). Deficits in cranial nerves III, IV, and VI can include disconjugate gaze (eyes do not move together), nystagmus (fine, rapid jerking eye movements), or ptosis (drooping of the eyelid). (Option 1) Cranial nerve II is the optic nerve and a sensory nerve. This nerve is assessed by testing the fields of vision for the client's ability to see objects in the field. In contrast to cranial nerves III, IV, and VI, the client does not track the object in the fields of vision, but instead keeps the eyes fixed and uses the peripheral vision to recognize objects or deficits in the field of vision. (Option 4) Cranial nerve V is the trigeminal nerve. The sensory portion of this nerve is assessed by testing sensation at the ophthalmic (forehead), maxillary (cheekbone), and mandibular (jaw line) branches by light touch. Corneal sensation is also a portion of the trigeminal nerve, but this is typically not tested by the nurse. Educational objective:Motor function of the eyes is tested by having the client track an object through the fields of vision and documenting any abnormalities in eye movement that represent cranial nerves III, IV, and VI. The health care provider is then notified of the findings. Additional Information Reduction of Risk Potential NCSBN Client Need

The nurse prepares to administer an IV infusion of potassium chloride through a peripheral vein to a client with hypokalemia. The health care provider's prescription states: IV potassium chloride 10 mEq (10 mmol)/100 mL 5% dextrose in water now, infuse over 30 minutes. What is the nurse's priority action? 1. Assess the patency of the peripheral IV site (24%) 2. Check the most current serum potassium level (33%) 3. Contact the health care provider to verify the prescription (39%) 4. Set the electronic IV pump to 100 mL/hr (2%) OmittedCorrect answer 3 39%Answered correctly

The recommended rates for an intermittent IV infusion of potassium chloride (KCl) are no greater than 10 mEq (10 mmol) over 1 hour when infused through a peripheral line and no greater than 40 mEq/hr (40 mmol/hr) when infused through a central line (follow facility guidelines and policy). If the nurse were to administer the medication as prescribed, the rate would exceed the recommended rate of 10 mEq/hr (10 mmol/hr) (ie, 10 mEq [10 mmol] over 30 minutes = 20 mEq/hr [20 mmol/hr]). A too rapid infusion can lead to pain and irritation of the vein and postinfusion phlebitis. Contacting the health care provider to verify this prescription is the priority action. (Option 1) The nurse would assess the IV site for swelling, tenderness, and redness just before initiating the KCl infusion and every 30 minutes during administration. However, this is not the priority action. (Option 2) The nurse would check the most current serum potassium level just before administering the KCl and may obtain another level following the infusion, if prescribed. This is not the priority action. (Option 4) An electronic IV pump should always be used to administer KCl. To administer the infusion at the recommended rate of 10 mEq/hr (10 mmol/hr), the nurse would set the pump at 100 mL/hr, but this is not the priority action. Educational objective:The maximum rate for infusion of IV potassium chloride through a peripheral vein is 10 mEq/hr, and the maximum rate through a central vein is 40 mEq/hr.

A nurse is assessing a client with type 2 diabetes mellitus who was recently started on pioglitazone. Which client data obtained by the nurse is most important to bring to the attention of the health care provider? 1. Bilateral pitting edema in ankles (22%) 2. Blood pressure is 140/88 mm Hg (3%) 3. Most recent HbA1c is 6.7% (9%) 4. Retinal photocoagulation in right eye (64%) OmittedCorrect answer 1 22%Answered correctly

Thiazolidinediones (rosiglitazone [Avandia] and pioglitazone [Actos]) are used to treat type 2 diabetes mellitus. These agents improve insulin sensitivity but do not release excess insulin, leading to a low risk for hypoglycemia (similar to metformin). These drugs can worsen heart failure by causing fluid retention and increase the risk of bladder cancer. Heart failure or volume overload is a contraindication to thiazolidinedione use. These medications also increase the risk of cardiovascular events such as myocardial infarction. (Option 2) The target blood pressure for a client with diabetes is <140/90 mm Hg. (Option 3) The goal HbA1c for diabetic clients is <7%. (Option 4) Diabetic retinopathy, a condition treated with retinal photocoagulation, is unrelated to thiazolidinedione use. If the client has a history of bladder cancer, then it should be reported. Educational objective:Thiazolidinediones (rosiglitazone [Avandia] and pioglitazone [Actos]) increase the risk of cardiovascular events (eg, mycoardial infarction, heart failure) and bladder cancer. Thiazolidinedione use increase insulin sensitivity but carries a low risk for hypoglycemia (similar to metformin).

A client on a medical unit recently received a diagnosis of end-stage renal disease and was told of the need to go on dialysis. This morning the client was found in the bathroom trying to commit suicide by hanging using hospital gown ties. The client was stabilized and transferred to the psychiatric unit. Which of the following is the highest priority nursing action for this client? 1. Assess the client's risk for another suicide attempt (21%) 2. Encourage the client to express current feelings about the medical diagnosis (7%) 3. Place the client in a private room near the nurses' station (0%) 4. Provide continuous one-to-one observation with the client (70%) OmittedCorrect answer 4 70%Answered correctly

This client has made a suicide attempt and is at high risk for additional suicidal behavior. Therefore, the client's priority need is for safety. The best nursing action is to provide one-on-one contact with the client to ensure constant observation and that the client does not engage in self-harm. The presence of the nurse will also convey a sense of acceptance, concern, and caring and provide an opportunity for the client to express feelings about the current situation. Additional nursing interventions for the client at high risk for suicide include the following: Removing sharp and other potentially harmful objects (eg, belts, metal eating utensils, ties, glass items) from the client's environment Making sure the client swallows medications Supervising the client during meals Placing the client in a semiprivate room near the nurses' station (to reduce social isolation and allow easier access to the client) Making rounds at irregular intervals for the client who does not need constant observation, as well as at shift changes and when staff is unusually busy Encouraging the client to express feelings, especially anger Having an open and honest conversation with the client about changing suicide risk (Option 1) This is an appropriate nursing action but not the highest priority action. (Option 2) This is an appropriate nursing action but not the priority action. (Option 3) A client at risk for suicide should be placed in a semiprivate room. Educational objective:The priority nursing action for a client who has made a recent suicide attempt is to ensure the client's safety. The best approach is to provide one-on-one contact and constant observation.

/An elderly client tells the nurse "I have experienced leg pain for several weeks when I walk to the mailbox each afternoon, but it goes away once I stop walking." What is the priority assessment the nurse should perform? 1. Assess for dry, scaly skin on the lower legs (4%) 2. Assess for presence or absence of hair growth on lower extremities (26%) 3. Check for presence and quality of posterior tibial and dorsalis pedis pulses (67%) 4. Obtain a dietary history (1%) OmittedCorrect answer 3 67%Answered correctly

This client is exhibiting symptoms of intermittent claudication or ischemic muscle pain that can be due to peripheral artery disease (PAD). PAD impairs circulation to the client's extremities. The nurse should first check for the adequacy of blood flow to the lower extremities by palpating for the presence of posterior tibial and dorsalis pedis pulses and their quality. Poor circulation to the extremities can place the client at increased risk for development of arterial ulcers and infection. The quality of circulation to the extremities will guide the treatment plan for this client; management will include risk factor modification for cardiovascular disease, drug therapy, and possibly surgical revascularization. (Option 1) Dry, scaly skin can be present in the client with PAD. It is a chronic condition of PAD and is not the priority assessment. (Option 2) When circulation to the extremities is impaired, the skin on the lower legs becomes thin, shiny, and taut; hair loss also occurs on the lower legs. This develops over time and would indicate that PAD has been present for a period of time and is not the priority assessment. (Option 4) The nurse should obtain a dietary history to assess for risk factors associated with cardiovascular disease. However, this is a lower level priority in this situation. Educational objective:The nurse caring for a client with intermittent claudication from PAD should assess the adequacy of circulation to the extremities by palpating and assessing the quality of posterior tibial and dorsalis pedis pulses. The quality of circulation will guide the treatment plan including risk factor modification, drug therapy, and possible surgical revascularization. Additional Information Physiological Adaptation NCSBN Client Need

//The nurse is caring for a client who is having a thoracentesis. Following the procedure, the nurse monitors for complications. The initial postprocedure monitoring plan should include what? Select all that apply. 1. Level of alertness 2. Lung sounds 3. Oxygen saturation 4. Respiratory pattern 5. Temperature 6. Urine output OmittedCorrect answer 1,2,3,4 37%Answered correctly

Thoracentesis is commonly used to treat pleural effusion. The health care provider (HCP) will prepare the skin, inject a local anesthetic, and then insert a needle between the ribs into the pleural space where the fluid is located. A complication of thoracentesis is pneumothorax, which occurs when the needle goes into the lung and causes the lung to slowly deflate, like a balloon with a small hole in it. Bleeding is another, yet less common, complication of the procedure. Signs of pneumothorax include increased respiratory rate, increased respiratory effort, respiratory distress, low oxygen saturation, and absent breath sounds on the side where the procedure was done (where the lung is collapsed) (Options 2, 3, and 4). Tension pneumothorax may also develop, with tracheal shift to the unaffected side, severe respiratory distress, and cardiovascular compromise. Altered level of consciousness may occur due to decreased oxygenation and blood flow to the brain (Option 1). A tension pneumothorax may be prevented by early detection of pneumothorax through appropriate monitoring. (Option 5) Infection would be a later complication (occurring a few days after the procedure), so monitoring temperature is not required during the initial postprocedure period. (Option 6) Urine output should not be affected by thoracentesis or the drugs administered for this procedure. Educational objective:Following thoracentesis, the nurse should monitor for signs of pneumothorax, including level of alertness, respiratory rate, respiratory effort, oxygen saturation, and lung sounds.

/After receiving report, which client should the nurse assess first? 1. Client on a heparin infusion with platelet count of 86,000/mm3 (86 x 109/L) (49%) 2. Client with dehydration with blood urea nitrogen of 24 mg/dL (8.57 mmol/L) (8%) 3. Client with myelodysplastic syndrome with white blood cell count of 2,000/mm3 (2.0 x 109/L) (22%) 4. Client with sickle cell disease with hemoglobin of 7.9 g/dL (79 g/L) and hematocrit of 24% (0.24) (20%) OmittedCorrect answer 1 49%Answered correctly

Thrombocytopenia is a serious complication of heparin products (eg, unfractionated heparin and low-molecular-weight heparin [eg, enoxaparin]). Regardless of its cause, thrombocytopenia usually results in bleeding complications. However, in heparin-induced thrombocytopenia (HIT) this usually leads to paradoxical venous and/or arterial thrombosis and less commonly to bleeding. The mechanism for thrombosis is unclear. The danger of HIT is risk of organ damage from local thrombi and/or embolization, leading to stroke and/or pulmonary embolism. HIT occurs over several days. The nurse should monitor platelet levels of clients on heparin and report a decrease of ≥50% from baseline or a drop below 150,000/mm3 (150 x 109/L) to the health care provider. If the client has HIT, all heparin products must be stopped immediately, and a different anticoagulant (eg, argatroban) should be started to prevent thrombosis risk. (Option 2) This client with mildly elevated blood urea nitrogen (normal 6-20 mg/dL [2.1-7.1 mmol/L]) needs IV fluids. This is not the priority. (Option 3) The client with myelodysplastic syndrome does not produce adequate blood cells. Low white blood cell count, platelets, and hemoglobin are expected. Although this client is at risk for infection, HIT has a higher priority. (Option 4) Decreased hemoglobin and hematocrit are expected in sickle cell disease due to chronic/acute hemolysis. A packed red blood cell transfusion may be needed, but this is not the priority. Educational objective:Drug-induced thrombocytopenia may result in bleeding; however, heparin-induced thrombocytopenia has an added risk of thrombosis. The nurse should monitor platelet levels of clients on heparin and report a decrease of ≥50% from baseline or a drop below 150,000/mm3 (150 x 109/L) to the health care provider. Additional Information Management of Care NCSBN Client Need

The nurse is interviewing a non-English-speaking client. Which best practices will the nurse use when working with a professional medical interpreter for clients of limited English proficiency? Select all that apply. 1. Address the interpreter directly 2. Ask the client's adult child to translate 3. Hold a pre-conference with the interpreter 4. Identify any gender or age preferences 5. Speak in short sentences OmittedCorrect answer 3,4,5 38%Answered correctly

Title IV of the Civil Rights Act of 1964 initiated national standards for appropriate care of culturally diverse clients. Clients with limited English proficiency have the right to receive medical interpreter services free of charge. When working with an interpreter, the nurse should apply the following best practices to maximize communication and understanding with the client: Address the client directly in the first person Speak in short sentences, pausing to allow the interpreter to speak (Option 5) Ask only one question at a time Avoid complex issues, idioms, jokes, and medical jargon Hold a pre-conference with the medical interpreter to review the goals of the interview (Option 3) Use a qualified professional interpreter whenever possible The nurse should avoid using interpreters from conflicting cultures (eg, Palestinian, Jewish) and be mindful of any cultural, gender, or age preferences (Option 4). (Option 1) The nurse should speak directly to the client, not the interpreter. (Option 2) A family member or friend may not have the vocabulary, knowledge, or skills to provide the best communication for the client. Untrained interpreters may omit or simplify critical pieces of information if they do not understand the terminology. Educational objective:When working with a medical interpreter, the nurse should apply best practices to maximize communication and understanding with the client. Key practices include speaking to the client directly; using short, simple sentences; avoiding the use of family members as interpreters; and being mindful of cultural, gender, or age preferences.

A client with hypokalemia is prescribed IV potassium chloride (KCl) to infuse at 10 mEq/hr. The pharmacy sends 20 mEq of KCl in 250 mL of D5W. To deliver the prescribed dose, the nurse sets the infusion pump at how many milliliters per hour (mL/hr)? Record your answer using a whole number. Answer: (mL/hr) OmittedCorrect answer 125 85%Answered correctly

Using dimensional analysis, use the following steps to calculate the infusion rate of potassium chloride (KCl) in milliliters per hour: Identify the prescribed, available, and required medication information Prescribed: 10 mEq KClhr Available: 20 mEq KCl250 ml D5W Required: mLhrPrescribed: 10 mEq KClhr Available: 20 mEq KCl250 ml D5W Required: mLhr Convert prescription to the infusion rate needed for administration Prescription×available medication=mLhrPrescription×available medication=mLhr OR (mEq KClhr)(mLmEq KCl)=mL KClhrmEq KClhrmLmEq KCl=mL KClhr OR ⎛⎝10 mEq KClhr⎞⎠⎛⎝250 mL20 mEq KCl⎞⎠=125 mL KClhr10 mEq KClhr250 mL20 mEq KCl=125 mL KClhr Educational objective:To calculate the infusion rate of potassium chloride, the nurse should first identify the prescribed dose (eg, 10 mEq/hr) and available medication (eg, 20 mEq/250 mL) and then convert to volume in milliliters per hour (eg, 125 mL/hr).

The health care provider prescribes 2 mEq (2 mmol)/kg of 8.4% sodium bicarbonate IV to be administered over the next 4 hours. The client weighs 150 lb, and the pharmacy supplies the following IV solution: 8.4% sodium bicarbonate in 1000 mL of D5W with 150 mEq (150 mmol) of sodium bicarbonate. At what rate in milliliters per hour (mL/hr) should the nurse set the infusion pump? Record your answer using a whole number. Answer: (mL/hr) OmittedCorrect answer 227 38%Answered correctly

Using dimensional analysis, use the following steps to calculate the infusion rate of sodium bicarbonate (NaHCO3) in milliliters: Identify the prescribed, available, and required medication information Prescribed: 2 mEq (2 mmol) NaHCO3kg/dose Available: 150 mEq (150 mmol) NaHCO31000 mL Required: mLhrPrescribed: 2 mEq 2 mmol NaHCO3kg/dose Available: 150 mEq 150 mmol NaHCO31000 mL Required: mLhr Convert prescription to infusion rate needed for administration Prescription×available data=mL per hourPrescription×available data=mL per hour OR (mEq (mmol) NaHCO3kg/dose)(kglb)(lb )(dosehr)(mLmEq (mmol) NaHCO3)=mL NaHCO3hrmEq mmol NaHCO3kg/dosekglblb dosehrmLmEq mmol NaHCO3=mL NaHCO3hr OR ⎛⎝⎜2 mEq (2 mmol) NaHCO3kg/dose⎞⎠⎟⎛⎝kg2.2 lb⎞⎠(150 lb )(dose4 hr)⎛⎝⎜1000 mL150 mEq (150 mmol)NaHCO3⎞⎠⎟2 mEq 2 mmol NaHCO3kg/dosekg2.2 lb150 lb dose4 hr1000 mL150 mEq 150 mmolNaHCO3 =227.2727¯ mL NaHCO3hr =227.2727¯ mL NaHCO3hr Round to a whole number 227.2727¯ mL NaHCO3hr=227 mL NaHCO3hr227.2727¯ mL NaHCO3hr=227 mL NaHCO3hr Educational objective:To calculate the hourly infusion rate of sodium bicarbonate, the nurse should first identify the prescribed dose (eg, 2 mEq [2 mmol]/kg/dose) and available medication (eg, 150 mEq [150 mmol]/1000 mL) and then convert to volume in milliliters per hour (eg, 227 mL/hr). Additional Information Pharmacological and Parenteral Therapies NCSBN Client Need

//The nurse caring for multiple clients who underwent renal system diagnostic testing should report which post-procedure finding to the health care provider? 1. 150 mL residual urine on bladder scan (50%) 2. Burning sensation when voiding after cystoscopy (22%) 3. Increased urinary output after arteriogram (14%) 4. Less than 10,000 organisms/mL on urine culture (12%) OmittedCorrect answer 1 50%Answered correctly

Various diagnostic tests, including bladder scans, urine cultures, cystoscopy, renal arteriograms, and renal scans, assess the renal system. It is necessary to understand the purpose and procedures for each examination when evaluating complications arising from these assessments. Portable ultrasonic bladder scanners are used at the bedside to determine the amount of residual urine in the bladder. Amounts >100 mL should be reported as the client may be experiencing urinary retention (Option 1). (Option 2) A cystoscope is inserted through the urethra to directly visualize the bladder wall and urethra. Irritation of the urethral and bladder lining from the insertion and manipulation of the cystoscope may cause a slight burning sensation with voiding for a day or two. (Option 3) Renal arteriogram is a radiologic test performed to visualize renal blood vessels to detect abnormalities (renal artery stenosis or aneurysm). A contrast medium is injected into the femoral artery; therefore, the client should be taught to increase fluid intake after the procedure to flush the dye from the body. Increased output is an expected finding. (Option 4) Urine is sterile, but the urethra contains bacteria and a few white blood cells. Less than 10,000 organisms/mL is a normal value for urine culture. Values >10,000 organisms/mL indicate urinary tract infection (UTI). Educational objective:Residual urine volume of >100 mL on bladder scan may indicate urinary retention. Urine culture showing values >10,000 organisms/mL can suggest UTI. Burning sensation is common after cystoscopy. Renal arteriogram is performed with a contrast agent; excretion of the dye with oral and intravenous hydration is recommended to prevent kidney injury.

The telemetry nurse is reviewing the cardiac monitors of 4 clients. Which cardiac rhythm is the priority for intervention by the nurse? 1. (1%) 2. (2%) 3. (71%) 4. (24%) OmittedCorrect answer 3 71%Answered correctly

Ventricular fibrillation (VF) is a lethal arrhythmia characterized by disorganized electrical activity in the heart ventricles. Because of this erratic electrical activity, the heart's muscles lose the ability to contract, resulting in loss of blood flow and pulse (eg, cardiac arrest). Nurses who identify a client with VF should immediately check the pulse, start CPR, and prepare the client for defibrillation (Option 3). (Option 1) Atrial fibrillation is a cardiac arrhythmia characterized by disorganized electrical activity in the atria and an irregular pulse rate. Clients may experience this condition chronically or in response to other medical conditions (eg, electrolyte imbalance). However, a client with VF has no pulse and is the priority for care. (Option 2) Premature ventricular contractions are abnormal electrical impulses in the ventricles that may occur spontaneously or in response to heart irritants (eg, stimulant medications, electrolyte alterations, pain). This arrhythmia is typically not harmful but requires monitoring by the nurse. (Option 4) Ventricular tachycardia, a potentially lethal dysrhythmia characterized by organized, rapid firing of electrical activity within the ventricles, may impair perfusion and often leads to cardiac arrest and/or VF. However, clients may have a pulse with ventricular tachycardia, making the client with VF and no pulse the priority. Educational objective:Clients with ventricular fibrillation, a lethal arrhythmia, require immediate treatment with CPR and defibrillation. A pulse may be present in ventricular tachycardia, so it should be addressed as soon as possible. Atrial fibrillation and premature ventricular contractions are pulsatile rhythms. Additional Information Physiological Adaptation NCSBN Client Need

The unlicensed assistive personnel (UAP) notifies the charge nurse that the client told the UAP that the client is feeling short of breath. What should the charge nurse do first? 1. Activate a rapid response team (0%) 2. Ask the UAP to take vital signs and report back (3%) 3. Notify the client's assigned licensed practical nurse (LPN) to assess the client (6%) 4. Personally go and auscultate the client's lungs (89%) OmittedCorrect answer 4 89%Answered correctly

When a registered nurse (RN) receives a report of a client complaint that is potentially ominous from a staff member of lesser qualifications, the RN should personally assess the client. This is the primary nursing assessment that will be used to decide if an urgent need exists and a change in the nursing plan of care is needed. (Option 1) It is important to assess the client prior to acting (the nursing process) on initiating a rapid response team based on a report from a staff member with less knowledge and skills. The nurse may not find an acute client need during the assessment. (Option 2) The UAP could be asked to go with the RN and obtain vital signs as the RN is assessing the client, but the UAP should not be asked to independently obtain this assessment first. (Option 3) The RN should assess the client personally rather than assign the task to the LPN. LPNs are assigned clients who are basically stable. The RN should be involved when there is new-onset potential instability in a client's condition. Educational objective:The nurse must assess the client personally (rather than delegating) when a potentially ominous report is made by a less-qualified staff member.

A health care provider (HCP) is screaming, "Why didn't you get surgery scheduled sooner!?," at the nurse in the hallway. People in the hallway are staring. What is the best initial reaction by the nurse? 1. Firmly indicate that the HCP cannot speak to the nurse in that manner (8%) 2. Immediately apologize and attempt to fix the situation (2%) 3. Say nothing and let the HCP vent frustrations (1%) 4. State that the conversation needs to take place in private and walk to a room (86%) OmittedCorrect answer 4 86%Answered correctly

When there is inter-staff disagreement, it is important to not have a public "show." The first action should be to take the conflict "off stage." This is especially true when there is a power/authority difference (eg, HCP/nurse). Rather than suggest and wait, the nurse should immediately leave and go to a private area. That way the disruptive person has to either follow the nurse or stop talking because there is no longer an audience. Once in private, the nurse can acknowledge the HCP's concerns and work to resolve the issue (Option 4). (Option 1) Confrontation and aggressive response usually do not resolve or diffuse the situation and will still involve an audience. (Option 2) The nurse should first take the conversation private as the HCP is not likely to calm down soon. The nurse can offer a blameless apology (eg, "I'm sorry there has been a problem") and then focus on the solution. This should occur out of the public eye. (Option 3) This response involves avoidance rather than working to resolve the situation. It does not benefit staff or clients to see providers having a public disagreement. Educational objective:The first response to public displays of disruptive behavior is to take action to make the conversation private.


Conjuntos de estudio relacionados

[HEALTH] Midterm #1, Health Assessment Midterm #2

View Set

IHO: Hypersensitivity Reactions (Dr. Nelson - 2 hrs)

View Set

Financial Reporting and Disclosures (F2) Missy

View Set

Technical Writing Skills for Support Professionals - Chapter 4

View Set